You are on page 1of 98

A Lngua Espanhola fcil?

Muitos brasileiros se recusam a estudar espanhol, por o considerarem um idioma fcil


e por se acharem capazes de entend-lo, sem nunca t-lo estudado. Ledo engano.
Embora o espanhol seja muito semelhante ao portugus e resulte mais fcil a um
brasileiro entend-lo do que entender o ingls ou o japons, este idioma possui muitas
diferenas e muitas palavras semelhantes com significados distintos que levam a erros
na compreenso e mesmo a ridculas gafes. Observe:
comum um turista brasileiro, que nunca estudou espanhol, perguntar em portunhol:
Aceita cartn de crdito?. Esta frase ser entendida como: Pe leo no papelo de
crdito?. Isto ocorre porque o verbo aceitar, em espanhol, significa pr leo, untar e
cartn significa papelo. O correto perguntar: Acepta tarjeta de crdito?, onde
acepta significa aceita e tarjeta quer dizer carto.
Exemplos de diferenas e semelhanas:
a) Palavras diferentes
Calle (rua); bolgrafo (caneta); coche (carro); goma (borracha); silla (cadeira); tiza
(giz); bombilla (lmpada); pizarra (quadro escolar) etc.
b) Palavras iguais ou semelhantes com significados diferentes (falsos cognatos)
Exquisito (saboroso); stano (poro); rojo (vermelho); largo (comprido); vaso (copo);
copo (floco); flaco (magro) etc.
c) Palavras semelhantes ou idnticas
Casa, hombre, mujer, estacin, fruta, gato, libro, pequeo etc.
Quanto do espanhol eu poderei entender sem estud-lo?
Uma pesquisa realizada pela professora Isabel Gretel Mara Eres Fernndez, da USP,
comprova que o brasileiro que nunca tenha estudado espanhol entende
aproximadamente 46% do espanhol falado e 58% do espanhol escrito. A pesquisa foi
feita com universitrios.
http://www.yspanus.com.br/idioma%20espanhol/fuja%20do%20Portunol.htm
TEXTO 2
Detrs del manejo de la lengua hay actitudes discriminatorias
Debate en Uruguay sobre el uso e hibridacin del lenguaje
Segn la lingista Graciela Barrios, cuando se dice que los jvenes deforman el
idioma, se est diciendo, en forma solapada: no me gustan los jvenes; cuando se dice
que los programas argentinos de televisin nos invaden y deforman el idioma, en
realidad lo que hay es una inseguridad lingstica y un complejo de inferioridad frente
al vecino poderoso; y cuando se ataca el portugol (mezcla de espaol y portugus) se
discrimina a las poblaciones de frontera.
El espaol es la lengua de 21 pases y de 350 millones de hablantes. Tiene una
unidad slida, al menos siete siglos de historia, y un prestigio literario universal. Pero
como toda lengua extendida a lo largo del tiempo y del espacio tiene diversas
variedades, algunas extintas y otras que conviven en la actualidad. Sin embargo, no
todas las variedades gozan de buena reputacin.
Desde hace unos meses se instal en Uruguay un debate cuyo centro est en la
forma que tienen sus habitantes de hablar espaol. El Ministerio de Educacin y Cultura

de Uruguay impuls, a travs de su titular, Yamand Fau, una campaa sin nimo
sancionatorio ni censurador en la que se llama a defender el idioma.
Pero la iniciativa fue impugnada por varios lingistas. En coincidencia con este
debate la lingista uruguaya Graciela Barrios, del Departamento de Lingstica de
la Facultad de Humanidades, est desarrollando una investigacin titulada Lengua
estndar y prescripcin idiomtica en Uruguay. Esta investigacin pretende detectar la
ideologa lingstica que hay detrs de las prescripciones estatales en poltica
lingstica.
Detrs de las polticas de dirigir la lengua hay actitudes discriminatorias,
comenta Barrios. Este tipo de debates, en realidad, son una excusa para decir otras
cosas, porque en ellos nunca se habla seriamente de los problemas de la lengua,
asegura la cientfica.
Barrios explic que la posicin de un acadmico de la lengua y la de un docente
de Espaol son naturalmente opuestas. El docente de Idioma Espaol tiene que ensear
la variedad estndar del espaol. Tiene que ensear a escribir, corregir la ortografa y
todas esas cosas. El nio no va a la escuela para que el maestro le diga que hable como
quiera. El docente de Espaol es un profesional de la lengua estndar. Pero para el
lingista la lengua estndar es una de las tantas variedades del idioma, ni mejor ni peor
que otras.
Barrios, que tambin asesora al instituto que forma a los profesores de la
educacin secundaria de Uruguay, entiende que en una sociedad como la nuestra, donde
existen ciertos requerimientos de comunicacin, es necesario ensear la variedad
estndar del espaol. En este tipo de sociedades hablar mal puede generar actitudes
negativas y determinar que alguien no consiga un trabajo. Si viviramos en una
sociedad homognea, donde las diferencias no existieran o no importaran,
otro sera el cantar. Pero hay que ver de qu manera se ensea el espaol: no es
adecuado ensear su variedad anacrnica, es necesario hacerlo de una manera funcional.
Tambin hay que preguntarse por qu a los adolescentes
les cuesta tanto aprender la variedad estndar. Es slo una falla del sistema
pedaggico? Uno habla la variedad con la que se identifica. En entrevistas que hicimos
recogimos respuestas como yo no voy a hablar como la concheta (presumida) de la
profesora de Idioma Espaol. Eso nos informa de que hay otros aspectos que importan
a la hora de hablar.
Las lenguas son, como las sociedades que las hablan, organismos vivos. Desde
que la Lingstica se constituy en una ciencia a partir de los cursos de Ferdinand de
Saussure, en la dcada de 1910 se sabe que la lengua tiene un doble carcter: el de
mantenerse estable y el de transformarse. Una generacin de determinada comunidad
hablar la misma lengua que su predecesor, pero su lengua ser diferente a la que hablen
los individuos de 10 generaciones posteriores. Los lingistas entienden que el espaol,
por ejemplo, es un estadio del latn, como lo son las otras lenguas romances: el italiano,
el portugus, el francs y el rumano. Desde este punto de vista, la tan mentada frase que
asegura que se est degradando el espaol es una falacia.
Qu espaol se debera hablar, el espaol en el que escribi Jorge Luis Borges
o el de Miguel de Cervantes, el que se hablaba en Castilla en el siglo pasado o en el que
escribieron los autores del siglo de oro espaol? Si un idioma se degrada, lo correcto
entonces sera hablar en latn, que tambin tiene sus diferentes estadios, o mejor an, en
su predecesor, el indoeuropeo?
A este fenmeno se suma otro. El corpus total de una lengua est diseminado por
entre todos sus hablantes y los textos que ellos producen y no en un individuo o un

grupo de ellos. Una lengua tiene distintas variedades y dialectos que tambin se
transforman a lo largo del tiempo y del espacio. Sin embargo, su transformacin no la
puede realizar una sola persona o 100 de ellas, ni siquiera 1.000. Cul es el espaol
correcto, el de Madrid, que difiere del de Barcelona, el de Montevideo, el de Bogot, el
de Buenos Aires, o el de las zonas rurales, donde la transformacin es ms lenta, de
cada uno de los pases hispanoparlantes? Uno es mejor que otro? Como si esto
fuera poco, la lenguas son permeables a la influencia de otras lenguas. En el espaol
presuntamente correcto hay miles de palabras que provienen del latn, del griego, del
rabe y de muchas otras lenguas. Desde este punto de vista, alarmarse por la invasin de
palabras del ingls o de cualquier otro origen es infundada, cuando no ingenua.
La posicin de la Sociolingstica es que no hay que sustituir la forma correcta
por la incorrecta, sino agregarlas. La idea es que el hablante est capacitado para
hablar de la manera adecuada segn las circunstancias de habla, no hay una sola forma
de comunicacin. El lenguaje es un hecho social y no hace ms que reflejar a la
sociedad, por eso hay tantas variaciones.
http://www.estrada.com.ar/institucional/docente/lecturas_1asp?IdSeccion=2
1&Idlec=13
ARTCULOS
Definidos e Indefinidos (Definidos e Indefinidos)
Assim como a lngua portuguesa, a lngua espanhola apresenta duas formas de artigo, o
definido e o indefinido, que indicam, respectivamente, que o substantivo se refere a algo
j conhecido, determinado ou j mencionado, ou que o substantivo se refere a algo no
conhecido ou no mencionado no texto.
Definidos

Indefinidos

Masculino
Singular

El

Masculino Singular Un

Masculino Plural

Los

Masculino Plural

Feminino Singular La
Feminino Plural

Las

Unos

Feminino Singular Una


Feminino Plural

Unas

Casos Particulares
1) obrigatrio o uso de artigo determinado para informar as horas, dias da
semana e datas.
Son las seis en punto.
El resultado de los exmenes saldr el lunes.

Nasc el 19 de febrero de 1982.


a) Diante dos nmeros que indicam as horas se usa artigo e se omite a palavra horas:
Son las siete.
b) Diante dos dias da semana se usa o artigo, sem preposio:
El domingo voy a la feria.
2) Diante de um nome de pessoa, pas, regio ou continente, no se usa o artigo, salvo
quando estiver determinado por um adjetivo, orao relativa ou complemento.
Espaa es un Estado de la Unin Europea.
La Italia del Norte es muy linda.
Excees: La Habana, La Argentina, La India, Los Estados Unidos, El Japn, etc.
3) Diante das formas de tratamento, exceto Don.
El general San Martn vino cenar conmigo.
La seora Mercedes duerme mucho.
Don Ricardo es muy guapo.
4) Emprega-se el no lugar de la e un no lugar de una diante de um substantivo feminino
singular iniciado por a ou por ha tnico para evitar cacofonia.
el agua (a gua) / el alma (a alma)
un guila (uma guia) / un hada (uma fada)
Quando o substantivo feminino estiver no plural, mantm-se a forma original:
las aguas / las almas / las guilas / las hadas
5) Artigos no precedem adjetivos possessivos, mas precedem pronomes. Em
portugus, para esse caso o uso do artigo facultativo.
Los mis libros. (errado) / Los mos. (correto)
O Artigo Neutro Lo (El Artculo Neutro Lo)

O artigo neutro lo, inexistente em lngua portuguesa, utilizado para substantivar


adjetivos e advrbios.
Lo mejor de todo fue la fiesta. (mejor = melhor - adjetivo)
La paz es lo ms valioso sentimiento. (ms = mais - advrbio)
Cuidado!
O artigo neutro LO utilizado antes de adjetivo + preposio. Se depois
do adjetivo no tiver preposio, usa-se o artigo definido masculino
singular EL.
Lo bonito en un partido es ver goles.
El bello coche de Pablo fue muy caro.
Tambm se utiliza diante do pronome relativo que. Equivale a aquilo que, o que.
Lo que me encanta en ti es tu inteligencia.
Atencin!
Nunca coloque o artigo neutro "lo" na frente de substantivos
masculinos. muito comum os brasileiros cometerem esse erro,
confundindo lo com o (artigo masculino, em portugus). Substantivos
masculinos aceitam somente o artigo el.
Expresses coloquiais com Lo
Emprega-se o artigo neutro lo em diversas expresses coloquiais. Por ele no
apresentar variao para gnero e nmero, chama-se neutro. Veja alguns exemplos:
Lo + adjetivo + que: intensifica o valor do
adjetivo.
No sabes lo complicada que es mi situacin en la oficina.
(Voc no sabe o quanto complicada minha situao no escritrio.)
Lo + advrbio + que: intensifica o valor do
advrbio.

No me di cuenta de lo rpido que pas este ao.


(No me dei conta do quanto passou rpido este ano.)
Lo + participio + que: intensifica o valor do
particpio.
Mira lo roto que est este traje!
(Olha que rasgada que est esta roupa!)
Contrao do Artigo (Contraccin del Artculo)
A lngua espanhola possui apenas dois tipos de contrao: al e del.
AL: Preposio a + artigo el
Voy al puerto.
DEL: Preposio de + artigo el
Vengo del puerto.
1) Rellenas las frases con las opciones correctas:
1) Estuve aqu _____ao pasado
2) Todas _____noches pasa un rato vendo la tele.
3) Hollywood, _____ centro de la industria cinematogrfica, lo es tambin de la
televisin.
4) A muchas mujeres no les gustan _____ labores de _____ cocina.
5) _____hombres suelen ser ms pesados que _____ mujeres.
a)el; las; el; los-la; los-las.
b)la; el; los; lo-la; las; los.
c)el; lo; la; las-los; los; las
d)las; el; lo; las-lo; las; las
e)el; los; los; el-la; los; las
2) Los artculos indefinidos correctos para las siguientes palabras son:

ABEJA JABN-CARACOL- LIMN-RELOJ

a)un, una, una, un, un


b)una, un, un, un, un
c)una, una, una, un, un
d)un, un, un, una, un
e)una, una, un, un, una
3) Rellena con el artculo que corresponda:
_______ vaca es _______ animal rumiante.
Mi casa tiene _______ puerta de madera.
______ familia de Juan es grande.
Pedro tiene ______ automvil nuevo.
______ avin sale a ____ nueve y media.
Brasil es ______ pas muy grande.
_______ verbo gustar no se conjuga como ________ otros verbos.
Tengo ____ caballo muy veloz.
______ artculos acompaan a ____ sustantivos.
_______ equipo de So Paulo es mejor que ______ otros.
_______ mujer paso por la calle.
4) Elija la opcin correcta:
1) Hoy es ________ da de mi cumpleaos.

a) el
b) la
c) una

2) Busco ________ gato negro que se llama Pookie.


a) uno
b) un
c) el
3) Quiero ir a ________ universidad de Miami.
a) una
b) el
c) la
4) Mis amigas Cristina y Sara son muy ________ .
a) divertidos
b) divertidas
c) divertida
5) Carlos y Violeta son ________ .
a) conservadores
b) conservadoras
c) conservadoros
6) Elena es ________ , verdad?
a) espaol
b) espaole
c) espaola
7) Es Alberto estudiante en ________ programa de matemticas o ciencias polticas?

a) el
b) la
c) uno
8) Me gusta mirar la tele todos ________ das.
a) las
b) unos
c) los
9) Creo que ________ educacin es importante.
a) el
b) uno
c) la
10) Jorge y yo somos muy ____________
a) serias y trabajadoras
b) serios y trabajadores
c) serios y trabajaderos
SUSTANTIVOS
Os substantivos so palavras variveis - possuem gnero masculino ou feminino - que
nomeiam os seres, pessoas, objetos, aes, lugares, sentimentos e estados. Alm do
gnero, podem variar de acordo com o nmero ou o grau.
Classificam-se em:
Prprios: Pablo, Per
Comuns: perro (cachorro), taza (xcara)
Concretos: puerta (porta), Juan
Abstratos: amistad (amizade), belleza (beleza)
Simples: ojo (olho), zapato (sapato)
Composto: pararrayos (pra-raios), econmico-social (econmico-social)

Primitivos e Derivados: tinta (tinta) e tintero (tinteiro)


Coletivos: rebao (rebanho), muchedumbre (multido)
Gnero dos Substantivos (Gnero de los Sustantivos)
No que diz respeito s pessoas e aos animais, o gnero dos substantivos pode ser
masculino ou feminino, sendo relacionado ao sexo (gnero natural).
1. So masculinos aqueles terminados em AJE e em AMBRE:
La playa de Copacabana tiene un paisaje esplndido!
Gabriel llevaba los dos rollos del alambre que pediste.
2. So masculinos as cores, os dias da semana, os meses e os nmeros:
El verde de tus ojos me encanta.
Todos los martes tengo clases de ingls.
El enero fue un mes de mucha nieve en Espaa.
Lo mejor de los atletas es el cuatro.
3. So femininos aqueles terminados em UMBRE:
Las costumbres de los japoneses son raras.
Exceo: el alumbre (a iluminao).
4. So femininos os nomes das letras:
la a, la be, etc.
1) As palavras abaixo no feminino so:
ABAD, REY, TESTIGO, SACERDOTE, CABALLO, LEN
a)
b)
c)
d)
e)

abadesa, reina, testigo, sacerdotisa, yegua, leona


abadesa, reyna, testiga, sacerdota, yegua, lena
abada, reyna, testiga, sacerdotisa, yegua, leona
abadeza, reyna, testigo, sacerdotisa, yegua, leona
abadesa, reina, testiga, sacerdotiza, yegua, leona

2) As palavras abaixo no masculino ficam:

GALLINA, OVEJA, PAVA, VACA, ACTRIZ, EMPERATRIZ


a)
b)
c)
d)
e)

pollo, carnero, pavn, boi, actor, emperador


gallo, carnero, pavo, buey, actor, emperador
gallo, carnero, pavn, boi, ator, emperador
pollo, carnero, pavo, buei, actor, emperador
gallo, chivo, pavo, buei, ator, emperador

Formao do Plural (Formacin del Plural)


a) A regra geral consiste em acrescentar um -s quando a palavra terminar em
vogal no acentuada e em ou .
casa - casas
moto - motos
caf cafs
domin domins
b) Adiciona-se es para pluralizar os substantivos que terminam em consoante, bem
como em vogais tnicas , .
emocin (emoo) - emociones (emoes)
jabal (javali) - jabales (javalis)
bamb (bambu) - bambes (bambus)
Atencin!
Os substantivos terminados em e aceitam tambm a terminao s para o plural
(jabals, bambs). No entanto, a forma que utiliza es considerada mais culta.
c) Acresenta-se -s maioria dos substantivos terminados em , mas a alguns so
acrescentados es.
sof - sofs
pap - paps
mam - mams
faral (babado) - faralaes (babados)
d) As palavras terminadas em z tm plural em ces:

pez

lpiz

peces

lpices

e) Os substantivos que terminam em s oux precedidos de vogal tona so invariveis,


ou seja, tm a mesma forma no singular e no plural.
la crisis (a crise) - las crisis (as crises)
el mnibus (o nibus) - los mnibus (os nibus)
el trax (o trax) - los trax (os trax)
Exceo: el nix (o nix) - los nices (os nix)
f) Os monosslabos que, no singular, terminam em s, sofrem acrscimo de es na forma
plural.
el mes (o ms) - los meses (os meses)
Observao:
Lembre-se que os artigos el e la se modificam ao assumirem a forma no plural,
tornando-se los e las.
el chico - los chicos
la seora - las seoras
Plural dos Substantivos Compostos
Descrio

Exemplo

A alterao ocorre no segundo


vocbulo

el sordomudo (o surdo-mudo)
los sordomudos (os surdos-mudos)

A alterao ocorre no primeiro


vocbulo

la situacin lmite (a situao-limite)


las situaciones lmite (as situaeslimite)
el pararrayos (o para-raios)

Invarivel

los pararrayos (os para-raios)


1) La formacin del plural de las palabras abajo estn correctas
en la siguiente alternativa:
MANIQU, BALN, MOTOR, SILLA, BAMB,FRANCS
a)
b)
c)
d)
e)

manequies, balons, motores, sillas, bambuses, franceses


manequines, balones, motores, sillas, bambs, franceses
maniquins,balones, motores, sillas, bambes, franceses
maniques, balones, motores, sillas, bambes, franceses
maniquins, balns, motors, sillas, bambs, francses

2) La formacin del plural de las palabras abajo estn correctas en la siguiente


alternativa:
SNTESIS, VIERNES, CRISIS, T, FE, PIE
a)
b)
c)
d)
e)

sntesis. viernes, crisis, ts, fes, pies


sinteses, vierneses, crisesis, teses, fes, pies
sintesis, viernes, crises, tes, fs, pies
sintesis, viernes, crises, ts, fes, pies
sintsisis, viernes, crises, ts, fs, pis

Sustantivos Heterogenricos
Ao tratarmos de conceitos e de seres inanimados, o gnero passa a ser determinado de
forma arbitrria (gnero gramatical). Devido origem comum do vocabulrio, o gnero
dos substantivos espanhis costuma ser o mesmo que em portugus, mas isso no
ocorre sempre. Nestes casos, chamamos estes substantivos de heterogenricos, pois
possuem um gnero em espanhol e outro em portugus.

Espanhol

Portugus

la baraja

o baralho

la costumbre

o costume

la cumbre

o cume

la sonrisa

o sorriso

la risa

o riso

la nariz

o nariz

la sal

o sal

la leche

o leite

la sangre

o sangue

la labor

o trabalho

la percha

o cabide

la alarma

o alarme

la coz

o coice

la crema

o creme

la paradoja

o paradoxo

la legumbre

o legume

la miel

o mel

la pesadilla

o pesadelo

la protesta

o protesto

la seal

o sinal

las gafas

os culos

Espanhol

Portugus

el ptalo

a ptala

el crter

a cratera

el maratn

a maratona

el humo

a fumaa

el estante

a estante

el guante

a luva

el estreno

a estria

el equipo

a equipe

el pantaln

a cala

el viaje

a viagem

el paisaje

a paisagem

el anlisis

a anlise

el dolor

a dor

el color

a cor

el origen

a origem

el puente

a ponte

el rbol

a rvore

el orden

a ordem

HETEROSEMNTICOS (FALSOS AMIGOS/COGNATOS)


!A Que nos Entendemos Perfectamente!

Desde Uruguay, el Sr lvarez, comerciante de bebidas y alimentos, llama a


Exportaces Brasilienses de Alimentao, en So Paulo Brasil.
Telefonista: Exportaciones de Alimentos, Bom da!
Sr.lvarez: Buenos das! Mire, soy Ricardo lvarez. Llamo desde Montevideo
y me gustara hablar con el seor Silva.
Telefonista:Um momento. Vou passar para Sonia, a secretria dele.
Sonia: Al?
Sr. lvarez:Buenos das! Habla usted espaol?
Sonia: No, mas eu entendo perfectamente. Pode falar vontade.
Sr. lvarez: Bueno, quera hablar con el seor Silva, pero si usted puede entender,
voy formalizar un encargo. Tengo varias tiendas de alimentos en Montevideo y
quiero concretar un negocio con su empresa. He odo decir que el servicio de ustedes
es de primera.
Sonia: Normalmente quem trata desses asuntos o senhor Silva, mas o senhor pode
falar comigo mesmo porque ele no est.Na ausencia dele eu resolvo qualquer coisa.
Ele foi pegar a mulher dele no aeropuerto.
Sr. lvarez Qu? ser bruto!
Sonia: Claro que no. Por que o senhor me pergunta uma coisa dessas?
Sr. lvarez: Usted no ha dicho que fue pegar a su mujer en pleno aeropuerto?
Sonia: Disse. E da?
Sr. lvarez: No, nada. All ellos! Yo slo quiero hacer negocios y no juzgar a la
gente, as que, vamos al grano.
Sonia: Pois no.
Sr. lvarez: Cmo que no? Usted no haba dicho que lo representaba?
Sonia: Mozo, estou dizendo que pode falar.
Sr. lvarez: Perdone Usted. Crea que usted haba dicho que no.
Sonia: Disse pois no e pois no em portugus quer dizer sim
Sr. lvarez: Madre ma! Qu confusin! Bueno a lo que me interesa. Como le
he dicho, quiero negociar con ustedes. Necesito algo que ha dado fama a su empresa
en todo el Mercosur. El nombre de Exportaciones Brasilienses est en la boca de
todos.
Sonia: Que bom!
Sr. lvares: Pues quiero una carga de chacinas y algunas toneladas de granada
Sonia: Meu Deus! O senhor louco? Esta uma empresa honesta. No lidamos con
essas coisas.
Sr. lvarez:Loca lo ser usted, seora! Cuelga el telfono.
ESPAOL PORTUGUS

ESPAOL

PORTUGUS

Pronto
Chacina
Pegar
Mala
Copo
Cachorro
Exquisito
Pastel
Paro
Presunto
Rato
Ratn
Regaar
Rojo
Rubio
Saco
Salada
Salsa
Seta
Sino
Zurdo

logo, acabado
carne de porco
bater
m
floco (de neve)
filhote
saboroso,gostoso
bolo
desemprego
suposto
momento
rato
resmungar
vermelho
loiro
casaco, palet
salgada
molho
cogumelo
mas,sim,seno
canhoto

Oso
Prestar
Granada
Engrasado
Oficina
Embarzada
Cuello
Sitio
Sobre
Sobrenombre
Solo
Stano
Suceso
Taller
Tapas
Tasa
Taza
Trozo
Vaso
Zueco
Rico

urso
emprestar
rom
engraxado
escritrio
grvida
pescoo
lugar
envelope
apelido
sozinho
poro
fato, acontecimento
aperitivos
taxa
xcara
pedao
copo
tamanco
delicioso

Ejercicios con los heterosemnticos, heterotnicos y heterogenricos


1) completa las oraciones con algunas de las palabras heterotnicas del recuadro.
Democracia, Atmsfera, elogio, Micrfono, burocracia, ciruga,
1. Necesitamos un _________________ para el orador que dar la conferencia.
2. La________________ interna hizo que las gestiones se demoraran varias semanas.
3. La ____________________ terrestre es la capa gaseosa que rodea a la Tierra.
4. El paciente vivir si se somete a una_________________ .
5. Una herramienta muy til para desarrollar la autoestima en los nios es el
______________.
6. En el siglo XIX, la ___________________ fue entendida especficamente como la
designacin de los gobernantes por medio del sufragio.
2) completa los huecos con los heterosemnticos adecuados del recuadro a
continuacin.
Pelado, embarazada, escoba, ratn, tazas, apellido, vaso, cachorros, zurdo, rato,
desnudo.
a) No entre ahora, estoy _____________.
b) El chico tena dificultades con la tijera, pues era ____________.

c) Mi __________es Hernndez y mi sobrenombre es Pli.


d) Los _______________de len que nacieron en el zoo son divinos.
e) Me puedes ayudar, por favor? No consigo hacer con que el __________de mi
computadora funcione.
f) Me compr ______________ nuevas para tomar el t.
g) La seora, que esta ___________________, sinti un exquisito perfume de
flores en las ropitas de su beb.
h) Para que le piso quede limpio, hay que barrerlo con una ___________.
i) Quiero un ____________ con agua.
3) Parece pero no lo es! Busca al intruso, o sea, marca con un crculo la palabra
de gnero opuesto a las dems.
a) precepto costumbre acuerdo contrato
b) color aprendizaje dolor nariz
c) huevo cerdo pavo legumbre
d) lunes sal miel leche
e) anlisis sangre coraje origen
4) Identifica las palabras cuyo sentido no corresponde y reescribe las frases
usando el vocablo adecuado.
a) Cuando el profesor descubri que estaba copiando qued embarazada.
b) Los talleres estn en rebaja.
c) Beba directamente de la garrafa.
d) Se quit toda la ropa y qued pelado.
e) El perro traa un oso en la boca.
f) Llev el coche a la oficina porque se rompi el limpiaparabrisas.
g) Aqu un copo de agua cuesta 10 euros.
f) Esa cantante ha tenido mucho suceso.
Substantivos Homnimos

So palavras com a mesma grafia que, ao mudarem de gnero, mudam tambm de


significado.
el clera (doena) / la clera (raiva)
el guarda (cobrador de nibus) / la guarda (tutela)
el polica (agente) / la polica (administrao)
el maana (futuro) / la maana (parte do dia)
Ojo!
Alguns nomes tem uma s forma para designar o masculino e o feminino, determinando
o gnero pelo artigo que se emprega.
el/la periodista (jornalista)
el/la turista (turista)
el/la cantante (cantor/a)
el/la atleta (atleta)
Grau dos Substantivos (Grado de los Sustantivos)
Os substantivos podem sofrer flexo de grau, dando a ideia de aumento (grau
aumentativo) ou de diminuio (grau diminutivo).
Para o grau aumentativo, o sufixo mais frequente on/ona.
tabla (tbua) - tabln (tabuo)
mujer (mulher) - mujerona (mulherona)
Tambm so usados os seguintes: azo/aza, ote/ota, acho/acha, achn/achona.
amigo - amigote (amigo)
rico - ricachn (ricao)
Para o grau diminutivo, o sufixo mais frequente no espanhol geral ito/ita.

libro (livro)

librito (livrinho)

Utilizam-se tambm: illo/illa, ico/ica, n/ina, uco/uca, uelo/uela. A escolha de um ou


outro no afeta a conotao que acrescentam s palavras, isto , as formas costumam
expressar a mesma idia: librito, librico, librillo.
Atencin!
- Nas palavras terminadas em n e r e nas palavras de duas slabas terminadas em e, o
sufixo aparece precedido por um c (cito/cita).
rinconcito (cantinho), pastorcito (pastorzinho), pobrecito (pobrezinho)
- Nos monosslabos e nas palavras de duas slabas que apresentam ditongo tnico, o
sufixo aparece precedido por ec (ecito/ecita).
florecita (florzinha), viejecita (velhinha)
ADJETIVOS
O adjetivo a palavra que funciona como modificador direto do substantivo,
qualificando-o. Concorda sempre com o substantivo que acompanha, sofrendo, assim,
variao de gnero, nmero e grau.
Variao de gnero: La camisa amarilla.
Variao de nmero: Los alumnos estudiosos.
Variao de grau: Victor es ms fuerte que Javier.
Classificao dos Adjetivos (Clasificacin de los Adjetivos)
Primitivos

Derivados

bueno (bom)

bondadoso (bondoso)

Simple

Compuesto

fuerte (forte)

multicolor (multicor)

Patrios ou Gentlicos
canadiense (canadense), chino (chins)

Gnero dos Adjetivos (Gnero de los Adjetivos)

a) Os adjetivos masculinos terminados em o ou e mudam a terminao para a na


formao do feminino.
feo (feio) - fea (feia)
grandote (grandalho) - grandota (grandalhona)
b) Nos adjetivos masculinos terminados em an, in, on, or e nos gentlicos terminados
em consoante, acrescenta-se -a na formao do feminino.
soador (sonhador) - soadora (sonhadora)
ingls (ingls) - inglesa (inglesa)
c) Os adjetivos invariveis mantm a mesma forma quando acompanham substantivos
masculinos ou femininos.
Un hombre feliz. (Um homem feliz.) - Una mujer feliz. (Uma mulher feliz.)
hermano menor (irmo menor) - hermana menor (irm menor)
Grau dos Adjetivos (Grado de los Adjetivos)
O adjetivo pode aparecer em trs graus: positivo, comparativo e superlativo.
Grau Positivo: o grau normal do adjetivo; quando este no expressa mais do que o
prprio sentido.
Los cuentos de Julio Cortzar son buenos.
Grau Comparativo: permite estabelecer relao de igualdade, inferioridade ou
superioridade entre as qualidades dos seres.
Comparativo de Superioridad
Juan es ms alto que Pablo.
Comparativo de Inferioridad
Mara es menos dedicada que Joana.
Comparativo de Igualdad
Este dulce es tan exquisito como aqul.

Para reforar os comparativos de superioridade e inferioridade se antepe a


forma mucho antes de mejor, peor, mayor, menor.
Salir con mi madre es mucho mejor que salir com mi hermana.
Grau Superlativo: expressa a qualidade do ser de forma intensa. Pode ser absoluto ou
relativo.
Superlativo Absluto: indica o grau mximo de qualidade. Ocorre por meio da adio
do sufixo simo/sima ao adjetivo, ou ainda pela anteposio de advrbios, tais como
muy, sumamente, etc.
Exemplo: triste.
Pablito est tristsimo / muy triste.
Superlativo Relativo: expressa qualidades de superioridade e de inferioridade no seu
grau mximo, mas com relao a outros nomes. Forma-se com as partculas ms /
menos precedidas pelos artigos el, la, los, las.
Exemplos: alegres e simptico.
Las ms alegres chicas llegaron.
Carlos es lo menos simptico de los hombres.
Observaes:
Alguns adjetivos alteram suas razes ao formar o superlativo absoluto por adio de
sufixo. Os superlativos abaixo so chamados irregulares:
Adjetivo

Superlativo Absoluto

amable (amvel)

amibilsimo

antiguo (antigo)

antiqusimo

fiel (fiel)

fidelsimo

libre (livre)

librrrimo

frtill (frtil)

ubrrimo

amigo (amigo)

amicsimo / amigusimo

pobre (pobre)

pobrsimo / pauprrimo

joven (jovem)

jovencsimo

Alguns adjetivos podem ter formas especiais para o superlativo:


Positivo

Comparativo

Superlativo

bueno (bom)

mejor (melhor)

ptimo / buensimo /
muy bueno / el mejor

malo (mau)

peor (pior)

psimo / malsimo / el
peor

pequeo (pequeno) menor (menor)

mnimo / pequesimo
/ el menor

grande (grande)

mayor (maior)

mximo / grandsimo /
el mayor

alto (alto)

superior (superior)

sumo / supremo /
altsimo

bajo (baixo)

inferior (inferior)

nfimo / inferior /
bajsimo

Ojo!
Em espanhol as formas ms pequeo e ms grande so corretas.
Este zapato me parece ms pequeo que mi pie.
Esta casa es ms grande que la ma.

1) Relacione las palabras de la columna de la izquierda con los contrarios de la


columna de la derecha.

grande

alegre

guapo

antiptico

barato

caro

gordo

pequeo

tonto

viejo

nuevo

feo

moderno

inteligente

sucio

antiguo

simptico

limpio

10 triste

delgado

LOS POSESIVOS
So aqueles que acompanham ou representam um nome, expressando uma relao de
posse. So variveis em gnero e nmero quando tomam a forma plena e na forma
apocopada so variveis apenas em nmero.
Formas tonas

Formas Tnicas

(Apocopadas)

(Completas)

mi (meu, minha)

mo, ma (meu, minha)

mis (meus, minhas)

mos, mas (meus, minhas)

tu (teu, tua)

tuyo, tuya (teu, tua)

tus (teus, tuas)

tuyos, tuyas (teus, tuas)


suyo, suya

su (seu, sua, dele, dela)

(seu, sua, dele, dela)

sus (seus, suas, deles, delas)

suyos, suyas
(seus, suas, deles, delas)

nuestro, nuestra *

nuestro, nuestra *

(nosso, nossa)

(nosso, nossa)

nuestros, nuestras

nuestros, nuestras

(nossos, nossas)

(nossos, nossas)

vuestro, vuestra *

vuestro, vuestra *

(vosso, vossa)

(vosso, vossa)

vuestros, vuestras

vuestros, vuestras

(vossos, vossas)

(vossos, vossas)
suyo, suya (seu, sua)

su (seu, sua)

suyos, suyas (seus, suas, deles,


delas)

sus (seus, suas, deles, delas)

As formas nuestro(a) e vuestro(a) e seus plurais no se apocopam.

Estos son mis padres, vuestros abuelos.


Atualmente as formas vuestro (s) e vuestra (s) so utilizadas apenas na Espanha. Os
hispano-americanos utilizam as formas suyo (s), suya (s).
Ese bolgrafo es vuestro. = Ese bolgrafo es suyo.
Las gafas son vuestras? = Las gafas son suyas?
Na orao, os possessivos podem exercer funo de adjetivo ou de pronome.
stos son mis materiales. (adjetivo)
El material mo est sobre la mesa. (adjetivo)
Este material es mo. (pronome)
Ateno!
Com base nos exemplos acima, note que:

- Se o adjetivo estiver anteposto ao substantivo, utilizam-se as formas tonas (mi, tu,


su, etc.).
- Se o adjetivo estiver posposto ao substantivo, utilizam-se as formas tnicas (mo,
tuyo, suyo, etc.).
- Quanto aos pronomes, utilizam-se sempre as formas tnicas (mo, tuyo, suyo, etc.).
1) Complete os dilogos com os adjetivos e pronomes possessivos em Espanhol.

a)De quin es ese bolso?


Es

. Lo estabas buscando. (mi)

b) Toma ste es
No, no es

. (tu)
. (mi)

c) Aquellas son

bicicletas? - (vuestro)

S, son

. (nuestro)

ch) Aquel libro es


S, es

. (mi)

d) Esta casa es
No, no es

? (tu)

? (tu)

, la

es de abajo.

e) Qu libros quieres leer el


Quiero el

. (su)

o el

? (mi, tu).

f) Aquellos son sus amigos?


No, no son

, son de mi hermano menor. (mi)

2) Complete os dilogos com os adjetivos e pronomes possessivos em Espanhol.


Tienes mi nmero de telfono?
S, tengo tu nmero de telfono.
Necesitas los mapas de Mara?
mapas.

S, necesito

Necesitas el dinero de tus padres?


S, necesito

dinero.

Tienes tu reloj nuevo?


S, tengo

reloj nuevo.

Necesitas mi direccin?
No, ya tengo

direccin.

Tienes el nmero del hospital local?


No, no necesito

nmero.

Deseas los libros de historia de Manuel?


No, no deseo

libros.

Estudias nuestro texto de ingls?


No, no estudio

texto.

Los demostrativos
I. Adjetivos demostrativos
Los adjetivos van delante del nombre al que determinan y concuerdan en gnero y
nmero con l. El espaol es ms preciso en relacin a la capacidad para indicar la
proximidad o bien la distancia, tanto en el tiempo como en el espacio, que otras lenguas
y esta idea de distancia viene dada esencialmente por los demostrativos. stos pueden
establecer una relacin temporal o espacial con el hablante.
1 grado 2 grado 3 grado

de
de
de
distancia distancia distancia
masculino este

ese

aquel

femenino esta

esa

aquella

masculino estos

esos

aquellos

femenino estas

esas

aquellas

singular

plural

adverbio
correspondiente

aqu / ac ah

all / all

1 grado de distancia: relativamente cerca de a la persona que habla.


Ejs.:
Este coche no me gusta nada. (espacio)
Este ao no voy a ir de vacaciones. (tiempo)
2 grado de distancia: ni cerca ni lejos, muchas veces se sita en funcin de los otros
dos.
Ejs.:
Esa carta me parece sospechosa, no hay que abrirla. (espacio)
Esa maana en la que todo ocurri yo no estaba. (tiempo)
3 grado de distancia: relativamente lejos de a la persona que habla.
Ejs.:
Aquel nio es el hijo de la directora. (espacio)
En aquella poca todava no nos conocamos. (tiempo)
II. Pronombres demostrativos
Los pronombres demostrativos sirven para nombrar y distinguir elementos que ya se
han mencionado anteriormente, pero sin repetirlos. La forma de esos pronombres
demostrativos varia segn el gnero y el numero, as como de los seres o las cosas que
representan. La funcin que ocupan en la frase no conlleva ningn cambio en su forma.
1 grado 2 grado 3 grado
de
de
de
distancia distancia distancia

masculino ste

se

aquel

singular femenino sta

sa

aquella

eso

aquello

masculino stos

sos

aquellos

femenino stas

sas

aquellas

neutro

esto

plural

adverbio
correspondiente

aqu / ac ah

all / all

Nota 1: Observar cmo el acento sirve para diferenciar el pronombre y el adjetivo.


Nota 2: Los adjetivos demostrativos acompaan siempre a un sustantivo, mientras que
los pronombres reemplazan un nombre.
Es por este motivo que no existe un adjetivo demostrativo neutro (en espaol no hay
ningn nombre neutro) y en cambio s un pronombre neutro.
Nota 3: Las formas neutras del pronombre en espaol son las siguientes:

Esto: Esto no puede continuar as.


Eso: Eso lo hago yo en cinco minutos.
Aquello: Qu es aquello que se ve detrs de las montaas?

(Hay que sealar que estas formas neutras nunca llevan acento)
1) Rellena con los demostrativos correspondientes:
Mi coche es viejo, pero _________ (lejos) coche es nuevo. Mi coche es viejo, pero
aquel coche es nuevo.
1. La casa blanca es antigua, pero __________ (cerca) casa es moderna.
2. Estas copas de vino son bonitas, pero ___________ (no tan lejos) son ms bonitas.
3. Estas naranjas estn maduras, pero __________ (lejos) son mejores .
4. Los helados de esta heladera no son buenos, pero los helados de ________ (lejos)
heladera son deliciosos.
5. Los mdicos de aquel hospital son muy buenos, pero los mdicos de ___________
(cerca) hospital son mejores.
6. La casa de mi hermano es grande, pero ________ (no tan lejos) casa es ms grande.

7. De quin son estas revistas? _____________ (cerca) revistas son de Luis y


_______(lejos) revistas son mas.
8. Este libro es para mi pap y _________(no tan lejos) libro es para mi mam.
Numerales Ordinales y Cardinales
Numerais Cardinais (Numerales Cardinales)
Os nmeros cardinais espanhis so muito semelhantes aos nossos. Abaixo segue uma
lista dos nmeros cardinais em espanhol.
0 cero
1 uno (un); una
2 dos
3 tres
4 cuatro
5 cinco
6 seis
7 siete
8 ocho
9 nueve
10 diez
11 once
12 doce
13 trece
14 catorce
15 quince
16 diecisis
17 diecisiete
18 dieciocho
19 diecinueve
20 veinte
21 veintiuno
22 veintids
23 veintitrs
24 veinticuatro
25 veinticinco
26 veintisis
27 veintisiete
28 veintiocho
29 veintinueve
30 treinta
31 treinta y uno(un/a)
32 treinta y dos
40 cuarenta
50 cincuenta
60 sesenta
70 setenta

80 ochenta
90 noventa
100 cien/ciento
200 docientos / as
300 trescientos / as
400 cuatrocientos / as
500 quinientos / as
600 seiscientos / as
700 setecientos / as
800 ochocientos / as
900 novecientos / as
1.000 mil
2.000 dos mil
100.000 cien mil
1.000.000 un milln
Algumas regras:
- Esses nmeros cardinais podem agir como substantivos.
Ex: Juan y Tomz fueram al parque. Los dos salieron muy alegres.
- Nos numerais, a conjuno y utilizada apenas entre as dezenas e unidades dos
nmeros compostos (Ex: trienta y dos); exceto pelos derivados do veinte (20) que so
escritos em uma nica palavra (Ex: veintitrs).
- Em espanhol, o nmeros dos (2) no muda de gnero como acontece no portugus. Ou
seja, no existe feminino para o mesmo. J para as centenas, existem formas masculinas
e femininas (doscientos doscientas/ quinientos quinientas).
- A forma ciento (100) utilizada quando ser seguida de unidades e dezenas, e nesse
caso, no se usa a conjuno y.
Ex: Ciento trienta; Ciento diecisis.
- A forma cien (100) utilizada quando equivaler ao cem ou acompanhar o mil e o
millones.
Ex: Cien dlares. Cien mil. Cien millones.
Com a forma milln e seu plural, millones, utilizado um numeral antes.
Ex: Cinco millones.
Numerais Ordinais (Numerales Ordinales)
So tambm muito parecidos aos nossos, com algumas pequenas diferenas.
1 primero
2 segundo
3 tercero

4 cuarto
5 quinto
6 sexto
7 sptimo
8 octavo
9 noveno
10 dcimo
11 undcimo
12 duodcimo
13 decimotercero
14 decimocuarto
15 decimoquinto
16 decimosexto
17 decimosptimo
18 decimoctavo
19 decimonoveno
20 vigsimo
30 trigsimo
40 cuadragsimo
50 quincuagsimo
60 sexagsimo
70 septuagsimo
80 octagsimo
90 nonagsimo
100 centsimo
200 ducentcimo
300 tricentsimo
400 cuadringentsimo
500 quincuagentsimo
600 sexcentsimo
700 septuagentsimo
800 octingentsimo
900 noningentsimo
1000 milsimo
Algumas Regras:
Na lngua falada atualmente, os nmeros ordinais so usados at o dcimo (10), e da
em diante, so usados os nmeros cardinais.
Os ordinais concordam com os adjetivos.
Ex: La tercera puerta est abierta.
Os ordinais primero (1) e tercero (3) sofrem perda da ltima letra, no caso a vogal o,
quando utilizados com substantivos masculinos no singular.
Ex: Tercer piso.
1) Escribe estos numerales en letras
1. 16

2. 4
3. 600
4. 56
5. 18
6. 31
7. 12
8. 23
9. 6000
10. 90
11. 25
12. 39
13. 625
14. 107000
15. 10
ADVERBIOS
O advrbio uma palavra que pode modificar um verbo, um adjetivo ou a outro
advrbio. sempre invarivel. Alguns, quando se referem ao substantivo, tomam
carter adjetivo. Os advrbios se dividem em:
Advrbios de Tempo (Adverbios de Tiempo)
ahora (agora)

mientras (enquanto) ** luego (depois)

anteayer (anteontem)

temprano (cedo)

entonces (ento)

maana (manh)

entretanto (enquanto isso)


**

anoche (ontem noite)

hoy (hoje)

an (ainda) *

pronto (em pouco tempo)

aun (inclusive) *

anteanoche (anteontem
noite)

despus (depois)

todava (ainda) *

ayer (ontem)

* todava = an (sinnimos) e diferente de aun.


** entretanto = mientras tanto (enquanto isso - sinnimos).
Advrbios de Modo (Adverbios de Modo)
apenas (apenas)

como (como)

bien (bem)

entonces (ento)

mejor (melhor)

inclusive (inclusive)

mal (pouco, insuficiente) slo (somente) *


peor (pior)

fcilmente (facilmente) **

as (assim)

* slo: somente / solo (adjetivo): sozinho


** e outros terminados em mente.Advrbios de Lugar (Adverbios de Lugar)
abajo (abaixo)

delante (diante)

alrededor (ao redor)

detrs (atrs)

arriba (acima)

ah (a) *

cerca (cerca, perto)

all (ali) *

lejos (longe)

aqu (aqui) *

* aqu: indica o lugar onde se encontra a pessoa que fala.


ah: designa um lugar mais prximo que all.
all: distante da pessoa que fala.
Advrbios de Quantidade (Adverbios de Cantidad)
casi (quase)

poco (pouco)

mucho (muito) *

muy (muito) *

ms (mais)

bastante (bastante)

menos (menos)

adems (alm disso)

* o advrbio muy usado diante de adjetivos e advrbios:


muy fcil (muito fcil)
muy lejos (muito longe)
* o advrbio mucho usado diante de substantivos e antes ou depois de verbos em
qualquer forma:
Tengo mucho trabajo.
l mucho ha viajado.
Excepcin!
Diante dos adjetivos mejor, peor, mayor e menor, e dos
advrbios ms, menos, antes e despus usamos o advrbio mucho.
Advrbios de Afirmao (Adverbios de Afirmacin)
ciertamente (certamente)

s (sim)

seguramente (com segurana) claro (claro)

Advrbios de Negao (Adverbios de Negacin)

jams (jamais)

nunca (nunca)

no (no)

tampoco (tampouco) *

No existe a forma tambin no para negar. Para isso, usa-se o tampoco.

Advrbios de Dvida (Adverbios de Duda)


acaso (caso/se)

quiz (s) (talvez) *

probablemente (provavelmente)

tal vez (talvez)

posiblemente (posivelmente)

* Quiz(s) se antepe ao verbo. Quando a palavra siguinte comea por -s, se usa a forma
quiz e no quizs. O verbo se conjuga no subjuntivo: Quiz salga.
Advrbios de Ordem (Adverbios de Orden)
antes (antes)

primeramente (primeiramente)

despus (depois) sucesivamente (sucessivamente)

A formao em mente:
Observe que o advrbio pode ser forrmado pelo acrscimo do
sufixo mente ao adjetivo feminino.
lenta - lentamente
Quando o adjetivo possui acento, ele o conserva.
fcil - fcilmente

1) Complete com os advrbios de quantidade em Espanhol.


a) Esto es
b) Tenemos
c) Necesitamos
ch) Quiero la
d) Esto es
e) Tengo
f) Slo tengo
g) Hay

para nosotros.(Demais)
dinero.(Pouco)
personas.(Muitas)
de tu premio.(Metade)
de lo que esperaba.(Mais)
diez euros en el bolsillo .(Apenas)
sobres.(Alguns)
mil personas en la fiesta de Omar.(Quase).

2) Relacione las palabras de la columna de la izquierda con los contrarios de la


columna de la derecha.

1 dentro

tarde

2 arriba

deprisa

3 delante

detrs

4 despacio

cerca

5 lejos

abajo

6 pronto

fuera

3) Elija el adverbio correcto:

. (correcta, correctamente)

a) Mario hace la carta


b) Rosa hizo

el pastel. (fcil, fcilmente)

c) Ellos hicieron

la tarea. (bueno, bien)

ch) T hiciste

la nota. (malo, mal)

d)Ella comi langosta


e) Nosotros estudiamos
f)

. (entonces, anoche)
en la escuela. (maana, ayer)

estaremos en casa. (Ayer, Hoy)

g) Ellos comieron
h) Los nios se durmieron

con mam. (temprano, maana)


. (tarde, nunca)

i) Yo

comer chile. (nunca, ayer)

j) T

estars en casa. (tarde, pronto)

k) Los maestros comieron,

nosotros jugamos. (mal, entonces)

CONJUNCIONES
As conjunes so palavras que unem dois termos de uma mesma orao ou duas
oraes. Estas oraes podem estabelecer uma relao de coordenao, ou seja, uma
est relacionada outra mas no h dependncia entre elas, ou estabelecem relao de
subordinao, ou seja, uma depende da outra para ter sentido completo.
Conjunes Coordenadas (Conjunciones Coordinadas / Coordinantes)
Copulativas
Unem termos ou oraes que expressam ideias similares, estabelecendo uma relao de
adio:
Ni rojo, ni morado; prefiero verde.
Tengo para desayunar pan y leche.
Quiero mi gaseosa con limn y hielo.
Cuidado!
A conjuno y muda para e quando a palavra que segue comea por i, hi, seguida de
consoante.
Este libro es facil e interesante.
Son padre e hijo.
Disyuntivas
Unem termos ou oraes que expressam ideias opostas, estabelecendo relao de
excluso:
Hay que tener dos o tres alumnos.
Cuidado!
A conjuno o muda para u quando a palavra que segue comea por o, ho.
Son siete u ocho?
Tu perro es mujer u hombre?
Quando a conjuno o aparece entre nmeros, deve ser acentuada para no ser
confundida com o nmero zero:

12 15.
Distributivas
Unem termos ou oraes que expressam diferenas lgicas, temporais, espaciais ou de
qualquer outro tipo:
Bien para m, bien para tu hermano, tendrs que contarlo todo. (bien... bien)
Ora por una cosa, ora por otra, nunca consigo estudiar. (ora... ora)
Ya en tren, ya en autobs, iremos igual. (ya... ya)
Uno para m, otro para t. (uno... otro)
Adversativas
Unem termos ou oraes que se contrapem entre si:
Me gustara ir, pero no tengo dinero. (= mas)
No quiero t sino caf solo. (mas sim)
No les gusta comer frutas sino manzanas. (exceto)
Esta chica no hace otra cosa sino llorar. (a no ser)
Saldr esta maana aunque llueva.
Tena muchos motivos para hacerlo hablar, sin embargo no lo hizo.
Outras conjunes que designam ideias contrrias: excepto, no obstante, antes, antes
bien, a pesar de, con todo, ms bien, fuera de.
Conjunes Subordinadas (Conjunciones Subordinadas / Subordinantes)
Causales
Expressam casua, motivo da ao expressa pelo verbo da orao principal:
La fiesta ser buena, ya que he invitado todos mis amigos.
Vamos sacar buenas notas en las pruebas porque estudiamos mucho.
Outras conjunes que designam causa: como, que, pues, puesto que, debido a que, etc.

Finales
Expressam objetivo ou finalidade da ao expressa pelo verbo da orao principal:
Lo har a fin de que entiendas.
Outras conjunes que designam finalidade: porque, para que, de modo que, etc.
Temporales
Expressam diferentes matrizes do tempo em que ocorre a ao expressa pelo verbo da
orao principal:
Mientras me bao, t haces las tareas. (enquanto - simultaneidade)
En cuanto lleguen los invitados, avsame. (to logo, assim que)
Te llamar apenas llegue a Madrid. (to logo, assim que)
Cuando era nia, te gustaba ir al cine?
Consecutivas
Expressam o efeito ou a conseqncia da ao expressa pela orao principal:
Tengo mucha hambre, conque comer unas galletas. (portanto)
No estudiaste lo suficiente, luego no tendrs buenas notas.
T eres la nica persona que ley el texto, as que eres quien lo puede explicar. (de
modo que)
Concesivas
Expressam concesso ou ainda uma oposio ideia expressa pelo verbo da orao
principal:
Aunque no lo merezcas, te ayudar. (embora)
Outras conjunes que designam concesso: a pesar de que, y eso que, si bien, etc.
Condicionales
Expressam condio necessria ou hiptese para que se realize a ao expressa pelo
verbo da orao principal:

Como me extraes mucho, te escribo.


Si buscas la paz, la encontrars.
Outras conjunes que designam condio: ya que, siempre que, con tal que.
1) Elije las conjunciones correctas:
1. Van a venir a la fiesta Mara
2. Hoy no queremos ir al cine
3. Aqul edificio tiene siete

(y, e) Irma.
(pero, sino) al teatro.
(o, u) ocho pisos.

4. Tengo muchos primos

(pero, sino) no veo a ninguno.

5. No me gusta el ftbol

(sino, pero) el basquetboll.

6. Yo vi la televisin
7. Ese carro es caro

(mientras, por lo tanto) planchaba.


(mientras, sin embargo) aqul es barato.

Pronombres interrogativos y exclamativos


Dividir a classe em grupos ou duplas. Cada grupo trar fotos de personalidades. O
primeiro grupo mostra a foto e faz perguntas para os demais grupos, que devero
responder, com frases exclamativas ou no.
Exemplos
Preguntas y Respuestas:
Quin es este hombre? Qu gran actor! Trabaja en el cine.
Qu hace esta mujer? Qu guapa! Es modelo internacional!
Dnde trabaja este nio? Qu voz tiene este muchacho! Trabaja en la radio FM.
Depois da atividade oral, anotem as perguntas e respostas em uma folha parte, se
atentando aos sinais de acentuao.
Preposiciones
Este coche es de mi padre. de: posesin
Me encantan las mesas de madera. de: materia
Los bancos abren de 8:30 a 14:00. de: horarios (de __ a __)
Este vino es de Jerez. de: origen, nacionalidad
En el armario hay una bolsa de patatas fritas. de: contenido
Quieres un poco de pastel? de: porcin, parte
Se muri de cncer. de: causa
De Barcelona a Madrid son ocho horas en coche. de: espacio, longitud
A mano, a mquina. a: modo (Cmo?)
Nos vamos a Madrid. a: lugar de destino: ir + a
A medioda, a medianoche. a: partes del da
De aqu a la plaza hay cien metros. a: lugar de destino (de ___ a ___)

Estoy buscando a Juan, lo has visto? a: complemento directo de persona


A los cinco aos entr en el colegio. a: edad
La clase de espaol empieza a las 9:00. a: hora exacta (A qu hora...?)
Al fondo, a la derecha/izquierda, al final. a: expresiones de lugar (Dnde...?)
La farmacia est a dos calles de aqu. a: distancia
Voy a clase tres veces a la semana. a: frecuencia
De lunes a viernes voy a clase de espaol. a: tiempo: de (principio) ________a (fin)
A m me encanta la msica espaola. a: complemento indirecto
Las cartas estn en la mesa. en: lugar (sobre, encima de...)
Las llaves estn en mi bolso. en: lugar (dentro de...)
Hizo el examen en 20 minutos. en: tiempo para hacer una cosa
En 1992/enero/Navidad/primavera estuve en Brasil. en: tiempo
(aos/mes/navidad/estacin)
Hay tres kilmetros desde la escuela aqu. desde: espacio: desde (principio)___hasta
(fin)
Trabajo en esta empresa desde junio. desde: origen en el tiempo (Desde cundo...?)
El testigo vio el accidente desde la ventana. desde: lugar, punto exacto
Ayer estudi desde las tres hasta las seis. desde: tiempo: desde (principio) ___ hasta
(fin)
Este regalo es para ti. para: persona de destino
Para los chicos, la comida espaola no es muy buena. para: opinin (para + nombre de
persona)
Vine a Espaa para estudiar espaol. para: finalidad (Por qu?)
Tengo que hacer esto para el lunes. para: tiempo exacto para terminar una cosa
Este tren va para Barcelona? para: lugar de destino
Mlaga est por el sur de Espaa, no? por: lugar no exacto, aproximado
Este tren pasa por la estacin sur? por: atravesar un lugar
Voy a clase tres veces por semana. por: frecuencia
Por la maana, por la noche, por la tarde. por: partes del da
Suspendi el examen por no estudiar. por: causa, razn, motivo
EXERCCIOS EXTRAS
1) Rellena los espacios en blanco con las preposiciones correctas:
Juan hoy se ha levantado enfermo. Tena que ir _____ trabajo, pero le dola la cabeza.
_____ su casa _____ la oficina hay diez kilmetros y _____ bicicleta es muy cansado ir
______ all. Ha llamado _____ su jefe para decirle que hoy no iba ______trabajar.
Despus, se ha vestido y ha bajado ______ la farmacia que est _____ dos calles de su
casa. Ha comprado unas aspirinas y ha vuelto ______ casa pero, cuando iba a abrir la
puerta, se ha dado cuenta de que sus llaves estaban dentro_____ casa, ____ la mesa de
la cocina. Como tena fro y no saba qu hacer, ha decidido coger la bici e ir a la
oficina, que all siempre hace calor.
2. Completa las siguientes frases con las palabras antes de, ante o delante.
a) ________tales argumentos no tuvo ms remedio que rendirse.
b) _________esa casa hay un quiosco de peridicos.
c) __________hablar con este seor, hable primero conmigo.
d) Le resulta un poco violento fumar _________ su padre.

e) __________nosotros se extenda un panorama desolador.


2. Completa las frases abajo con las palabras para, hacia, con, sin o contra.
a) Vente _________casa a ver la televisin.
b) Han salido _______ Valencia hace dos das.
c) _______mi secretaria estoy perdido.
d) _______todo pronstico, ha perdido el Madrid _______el Gijn.
e) Crrase un poco __________ la izquierda.
f) Est usted en ________ o a favor del rgimen?
1) Seleccione la opcin correcta:
1. El coche gir y se dirigi ... la plaza.
a) hacia
b) hasta
2. He comprado este disco ... Pepe.
a) para
b) por
3. He dejado las llaves ... la mesa
a) sobre
b) entre
4. Estuve trabajando ... la medianoche.
a) durante
b) hasta
5. Mi casa est ... la boca del metro.
a) en
b) junto a
6. La carne estaba tan dura que no se poda cortar ni ... el cuchillo.
a) con
b) por
Los pronombres indefinidos
El pronombre indefinido reemplaza al adjetivo indefinido y el sustantivo
que aqul modifica. No describe ninguna caracterstica del sustantivo, sino
que slo indica cantidad

Ellos son:

Los pronombres indefinidos


mucho, muchos; mucha, muchas
poco, pocos; poca, pocas
tanto, tantos; tanta, tantas
bastante, bastantes
demasiado, demasiados; demasiada, demasiadas
alguno, algunos; alguna, algunas
ninguno; ninguna
algo, nada

Ejemplos:

Cunto dinero tienes? No tengo mucho.


Cunta agua queda? Creo que hay poca.
l ha tenido cinco esposas; ha tenido demasiadas.
Vendrn algunos? No creo que venga ninguno.
Hay algo en el bolso? No hay nada.

El pronombre indefinido ninguno/ninguna no se


emplea en forma plural.

1) COMPLETAR CON PRONOMBRES (algo, alguien, cualquiera, alguno,


ninguno, nadie, quin...)

1.

suspendi el examen.

2.

entr en su casa mientras dorma.


.

3. Entr en la tienda, pero no compr


4. Sali sin que

le viese.

5. Eso lo puedes hacer en


6. No mostr
7. Eso es

banco.

inters por acabar la conversacin.


que no puedo entender.

8. A pesar de la huelga no falt


9. Ya no haba
10.Escondi

a clase..

duda, haba aprobado el examen.


en su bolso antes de salir.

Pronombre Personal Complemento


So aqueles que exercem funo sinttica de objeto direto, objeto indireto ou pronome
reflexivo. Aparecem nas seguintes formas:
tonas: No precedidas de preposio.
Tnicas: Sempre precedidas de preposio.
FORMAS TONAS: me / te / se / nos / os / se
As formas tonas so empregadas nos seguintes casos:
a) Na conjugao de verbos reflexivos
Esteban se lava.
Clzate los zapatos.
Maana me despierto a las diez.
b) Como complemento direto ou indireto
No nos llames tan tarde. (objeto direto)
Me mandaron un regalo. (objeto indireto)
c) Como expresso de involuntariedade

Utiliza-se quando a ao no ocorre por interveno deliberada de um agente, mas


acidentalmente. Usa-se se + pronome tono + verbo na 3 pessoa (concordando com o
substantivo).
Cuidado! El perro se te est escapando!
Se me rompi la taza.
Se nos cayeron los relojes.
FORMAS TONAS: lo / la / los / las
Empregam-se sempre como complemento direto, ou seja, substituindo objetos diretos.
1) Llama un taxi, por favor.
Llmalo, por favor.
2) Visitar a mi familia en mis vacaciones.
La visitar en mis vacaciones.
3) Has encontrado a tus amigos?
No, los estoy buscando.
Ateno!
Diferentemente do portugus, em espanhol os objetos diretos
de pessoa ou coisa/animal personificado aparecem precedidos
da preposio a. Repare nos exemplos dados acima: os
objetos diretos precedidos da preposio a (segundo e terceiro
exemplo) so objetos de pessoa, enquanto que o objeto direto
no preposicionado (primeiro exemplo) refere-se coisa no
personificada.
1) Sustituya los sustantivos que sirven como complemento directo e indirecto con
sus respectivos pronombres. A menos que usted desee lo contrario, es preferible
que construya sus oraciones utilizando el presente de indicativo.
Modelo
Marta y Antonio/traer/las
bebidas/a nosotros

Marta y Antonio (Ellos)


nos las traen.

1. yo/enviar/la carta/a Juan


______________________________________________________
2. usted/entregar/el auto/a la chica
_______________________________________________________
3. vosotros/ofrecer/bebidas/a los invitados
______________________________________________________
4. t/derrochar/electricidad
______________________________________________________
5. Pepe/no dar/la solicitud/a sus amigas
_______________________________________________________
6. ustedes/beber/el agua
________________________________________________________
7. el detective/entregar/la evidencia/al juez
________________________________________________________
8. nosotros/elegir/el curso/para ti
_________________________________________________________
9. los estudiantes/buscar/los papeles/para la maestra
__________________________________________________________
10. Susana/traer el informe/para m
___________________________________________________________
11. el camarero/servir/el flan/a los nios
____________________________________________________________
12. ellos/servir/la cena/a las chicas
____________________________________________________________
13. Berta y yo/pedir/la cuenta/al mesero

_____________________________________________________________
14. ustedes/decir/mentiras/a nosotros
______________________________________________________________
15. t/traer/el pauelo/a Matilde
______________________________________________________________
PRESENTE DE INDICATIVO
Conjugacin tiempo PRESENTE
Los verbos en espaol se agrupan en tres series o conjugaciones, de acuerdo con la
terminacin del infinitivo:
Primera conjugacin
Ar
Amar

Cantar

Bailar

Segunda conjugacin
Er
Comer

Vender

Tercera conjugacin
Ir

Beber

Partir

Sentir

Surtir

MODO INDICATIVO TIEMPO: PRESENTE


PRONOMBRES

Eu
Tu
Ele

PARA A PRIMERA
CONJUGACIN

PARA A SEGUNDA
CONJUGACIN

-o
-as
-a

-o
-es
-e

Yo
T
l

PARA A
TERCERA
CONJUGACIN
-o
-es
-e

Nos Nosotros
-amos
-emos
-imos
Vos Vosotros
-is
-is
-s
Eles Ellos
-an
-en
-en
Entonces, en el presente de Indicativo, para todos los verbos regulares, la raz ser
aquella que corresponde al verbo y la desinencia ser siempre la misma que aparece en el
cuadro.

USOS DEL PRESENTE


Presente actual: indica una accin que incluye el tiempo presente. Esta accin puede ser
puntual o durativa.
Ejemplos: En este momento, el ganador recibe el trofeo. (puntual)
La clase se realiza con normalidad. (durativa)
Presente habitual: accin no continua.
Ejemplo: Los domingos voy a misa.

Presente atemporal: accin que se desarrolla fuera del tiempo; se usa en sentencias,
refranes y definiciones cientficas.
Ejemplo: Los carnvoros son animales vertebrados.
Presente histrico: hace referencia a acciones pasadas, situndolas en una perspectiva
ms cercana.
Ejemplo: Cristbal Coln descubre Amrica en 1492.
Presente con valor de futuro: anticipa acciones futuras; habitualmente se acompaa de
referencias temporales futuras.
Ejemplo: El prximo viernes tengo clases de espaol.
Presente
imperativo:
Ejemplo: T te callas!

se

utiliza

para

dar

rdenes.

1) Elije el verbo conjugado correctamente en presente de indicativo:


1. Los amigos de la familia (intentar) .................... consolar a la desamparada viuda.
a

intento

intentan

intientan
2. Los enamorados se (besar) .................... en el parque.

besan

besas

biesan
3. Nosotros casi nunca (beber) .................... vino en las comidas, slo agua!

bebis

biebemos

bebemos
4. Usted (ignorar) .................... quizs quin soy yo!!

ignoris

ignora

ignuera
5. Yo (poseer) .................... una inmensa biblioteca de libros medievales.

posea

poseyo

poseo
6. Mi padre (coger) .................... el tranva todos los das.

coje

coge

cuoge
7. Ricardo (escuchar) .................... siempre el mismo disco.

escucha

escuecha

hescuca
8. (Nosotros) nos (lavar) .................... los dientes tres veces al da.

lavis

lavamos

lavaramos

9. Qu testarudos son tus hijos!: Se (obstinar) .................... en hacer las cosas a su


manera!
a

obstinan

obstienan

obstinis
10. Julio Iglesias (vivir) .................... en Miami.

vive

viva

vivo
11. T (leer) .................... el peridico todos los das?

leas

leyes

lees
12. Por qu (vosotros) os (vengar) .................... sobre unos pobres inocentes?

venga

vengis

viengis
13. Por qu (escribir) .................... (ustedes) una carta al Presidente del Gobierno?

escriben

escriban

escrieben

14. Con este cambio de dirigentes, nuestros negocios (emprender) .................... un


nuevo rumbo.
a

emprendas

emprienden

emprenden

15. No me (importar) .................... si es alguien importante o un perfecto


desconocido!
a

importo

impuerto

importa
16. Margarita (echar) .................... lea al fuego.

hecho

echa

hecha
17. Este pasadizo (comunicar) .................... la antigua biblioteca con el jardn.

comunico

comunique

comunica
18. "Quien (cantar) ...................., su mal (espanta) ...................."

canta

canto

cantara
19. Sara (abrir) .................... la tienda a las nueve y media.

abras

abra

abre
20. T me (reprochar) .................... que siempre lo mismo.

reprochas

repruechas

reproche
PRESENTE DE SUBJUNTIVO

Conjugacin tiempo PRESENTE


Los verbos en espaol se agrupan en tres series o conjugaciones, de acuerdo con la terminacin
do infinitivo:
Primera conjugacin
Ar
Amar

Cantar

Bailar

Segunda conjugacin
Er
Comer

Vender

Beber

Tercera conjugacin
Ir
Partir

Vivir

Surtir

MODO INDICATIVO TIEMPO: FUTURO


PRONOMBRES

Eu

Yo

Tu
Ele
Nos
Vos

T
l
Nosotros
Vosotros

Eles

Ellos

PARA LA
PRIMERA
CONJUGACIN
-e

PARA LA
SEGUNDA
CONJUGACIN
-a

PARA LA
TERCERA
CONJUGACIN
-a

-es
-e
-emos
-is

-as
-a
-amos
-is

-as
-a
-amos
-is

-en

-an

-an

Entonces, en el Presente de Subjuntivo, para todos los verbos regulares, la raz ser aquella
que corresponde al verbo y la desinencia ser siempre la misma que aparece en el cuadro.
1) Completa las frases conjugando los verbos entre parntesis.
1. Es posible que

(hablar, l) ingls.

2. Es importante que

(escuchar, vosotros) cuando hablo.

3. Es necesario que

(trabajar, t) hasta tan tarde?

4. Es imprescindible que
5. Es posible que os

(vivir, vosotros) cerca de aqu.


(visitar, yo) en Navidades.

6. Es conveniente que

(comer, nosotros) antes de irnos.

7. No es necesario que

(acabar, yo) el proyecto hoy.

8. Es posible que

(comprar, ellos) otro piso.

9. Es probable que no

(pronunciar, ella) bien esa palabra.

10. No es imprescindible que

(bajar, t) la basura esta noche.

Verbos regulares
Comprar

Imperativo Positivo

Imperativo
Negativo

(yo)

(t)

compra

no compres

(usted)

compre

no compre

(nosotros)

compremos

no compremos

(vosotros)

comprad

no compris

(ellos)

compren

no compren

Verbos con diptongo y verbos con cambio de vocal:

PENSAR: pienso
Pensar

Imperativo Positivo

Imperativo
Negativo

(yo)

(t)

piensa

no pienses

(usted)

piense

no piense

(nosotros)

pensemos

no pensemos

(vosotros)

pensad

no pensis

(ellos)

piensen

no piensen

Servir

Imperativo Positivo

Imperativo
Negativo

(yo)

(t)

sirve

no sirvas

(usted)

sirva

no sirva

(nosotros)

sirvamos

no sirvamos

(vosotros)

servid

no sirvis

(ellos)

sirvan

no sirvan

SERVIR sirvo

Verbos irregulares:
Los verbos irregulares parten de la primera persona singular
del presente indicativo (yo hago) para formar el imperativo formal y el imperativo
negativo (formas que corresponden al SUBJUNTIVO). El imperativo de t muchas
veces es una forma apocopada: haz y no *hace). El imperativo afirmativo de vosotros
se forma como siempre, cambiando la r del infinitivo por la d
HACER hago
Hacer

Imperativo Positivo

Imperativo
Negativo

(yo)

(t)

haz

no hagas

(usted)

haga

no haga

(nosotros)

hagamos

no hagamos

(vosotros)

haced

no hagis

(ellos)

hagan

no hagan

Formas apocopadas del imperativo de t:


decir

hacer

poner

salir

ser

tener

venir

di

haz

pon

sal

ten

ven

El imperativo de IR (muchas veces el verbo IR se usa en su forma reflexiva)


Ir

Imperativo Positivo

Imperativo
Negativo

(yo)

(t)

ve, vete

no (te) vayas

(usted)

vaya, vyase

no (se) vaya

(nosotros)

vayamos, vaymonos no (nos) vayamos

(vosotros)

id, idos

no (os) vayis

(ellos)

vayan, vyanse

no (se) vayan

El imperativo de SER

Ser

Imperativo Positivo

Imperativo
Negativo

(yo)

(t)

no seas

(usted)

sea

no sea

(nosotros)

seamos

no seamos

(vosotros)

sed

no seis

(ellos)

sean

no sean

El imperativo de SABER
Saber

Imperativo Positivo

Imperativo
Negativo

(yo)

(t)

sabe

no sepas

(usted)

sepa

no sepa

(nosotros)

sepamos

no sepamos

(vosotros)

sabed

no sepis

(ellos)

sepan

no sepan

1) Completa las frases conjugando los verbos en imperativo afirmativo.


1. Juan! (Cerrar, t)
2. (Esperar, vosotros)

la puerta.
aqu. Ahora vuelvo.

3. (Recoger, vosotros)

vuestras cosas que nos vamos.

4. (Hablar, t)

ms alto, por favor. No te oigo.

5. (Indicar, t)

la respuesta correcta.

6. (Completar, vosotros)
7. (Conjugar, vosotros)
8. (Subir, t)

los verbos en pretrito simple.


el volumen de la televisin, por favor.

9. (Corregir, vosotros)
10. (Comentar, t)

las frases.

los ejercicios.
el siguiente poema de Borges.

2) Completa las frases conjugando los verbos en imperativo negativo.


1. No

(traer, t) animales a casa.

2. No

(sentarse, vosotros) ah. Est ocupado.

3. No

(hacer, ustedes) tanto ruido.

4. No me

(volver, t) a llamar.

5. No

(separarse, t) de m.

6. No

(hablar, vosotros) con la boca llena.

7. No

(seguir, t) molestando a tu hermana.

8. No

(entrar, vosotros) sin llamar.

9. No

(preocuparse, ustedes) . Todo saldr bien.

10. No

(ponerse, nosotros) nerviosos. No es tan grave.

Verbo Gustar
1) Completa con el pronombre y el verbo
a)
b)
c)
d)
e)
f)
g)
h)
i)
j)
k)
l)
m)
n)
o)
p)
q)
r)
s)
t)
u)
v)
w)
x)
y)

A Luis no __________________mucho el ftbol.


A Rosa ______________ andar por el campo.
A nosotras no ____________salir por la noche.
A mis hijos _____________ leer.
A ellos __________ los dulces
A m _____________ ver la tele
A ellos _____________ mucho las pelculas
A mi madre ____________ leer biografas
A ti ____________ los animales?
A ella ______________ tomar sol
A l ____________ los videojuegos
A nosotras ______________ navegar por internet
A los chicos ______________ jugar al ftbol
A Mariana ____________ mucho ir al cine
A m ______________ el ordenador
A ti ______________ bailar
A ustedes ______________ viajar
No me _____________ su coche nuevo
Me __________ ir de compras
Le ______________ su trabajo
Te ___________ estudiar? A m s
A m no _______________ la cebolla. A m tampoco
A m _____________ mucho el color rosa. A m tambin
A l ______________ trabajar mucho. A m no
A ellas ____________ hacer ejercicios fsicos. A nosotras tambin
_____________.
z) Cul es la ropa que ms ______________?
2) Contesta a las preguntas personales:
a)
b)
c)
d)
e)
f)
g)
h)
i)

Cul es la comida que ms _____________?


Cul es el color que ms _______________?
Cul es el estilo de ropa que ms ____________?
Cul es el/la amigo(a) que ms ______________?
Cul es la asignatura que ms ______________?
Cul es el festivo que ms ________________?
Cul es la gaseosa o zumo que ms _______________?
Cul es el da de la semana que ms _______________?
Cul es el deporte que ms ____________?

Apcope
Apcope a perda de uma letra ou slaba no final de algumas palavras. Isto acontece
com alguns advrbios e adjetivos quando esto diante de um substantivo, numeral,
advrbios ou adjetivos.
Adjetivos
Os adjetivos cualquiera (qualquer) e grande (grande) sofrem apcope quando
estiverem diante de um substantivo no singular masculino ou feminino.
Ex.:
- Juan llegar en cualquier momento.
- Es una gran fiesta para ella.
Quando sofrem apcope:
- Cualquiera passa a ser Cualquier;
- Grande passa a ser Gran.
Tambm sofrem apcope os adjetivos Bueno (bom), malo (mal), primero (primeiro),
tercero (terceiro), uno (um), alguno (algum) e ninguno (nenhum) quando estiverem
diante de um substantivo masculino no singular.
Ex.:
- Tomz es un buen mdico.
- Mi padre es un mal cantor.
- Juan fue el primer chico a ganar la medalla.
- Estoy en mi tercer ao de estudios.
- Carmen est con algn libro.
- Yo no necesito ningn consejo de l.
Quando sofrem apcope:
- Bueno passa a ser Buen;
- Malo passa a ser Mal;
- Primero passa a ser Primer;
- Tercero passa a ser Tercer;
- Uno passa a ser Un;
- Alguno passa a ser Algn;
- Ninguno passa a ser Ningn.
O adjetivo Santo (santo) sofre apcope quando diante de nomes prprios.
Ex.: San Pedro, San Juan, San Miguel, etc..
Porm h algumas excees: Santo Toms, Santo Tom, Santo Toribio, Santo
Domingo.

Quando sofre apcope:


- Santo passa a ser San.
Numerais
Os numerais Ciento (cem) e Veintiuno (vinte e um) sofrem apcope diante de um
substantivo feminino ou masculino (este tambm no plural).
Ex.:
- Ella dice que haban unas cien personas en lo baile.
- Ello cumpli veintiun ans.
Quando sofrem apcope:
- Ciento passa a ser Cien;
- Veintiuno passa a ser Veintin;
Advrbios
Sofrem apcope os advrbios Tanto (tanto) e Cuanto (quanto) quando diante de
adjetivos ou advrbios.
Ex.:
-Juan est tan cansado.
Quando sofrem apcope:
- Tanto passa a ser Tan;
- Cuanto passa a ser Cuan.
Lembre-se:
Simples a forma plural referente a simple (singular).
una idea simple (uma ideia simples) - unas ideas simples (umas ideias simples)
1) Neste exerccio voc dever completar as frases com as palavras que esto entre
parnteses. Em algumas palavras ocorrero apcope e outras no.

a) Fernando Alonso es un
b) Le deseamos a todos un
c) Tengo una
ch)

piloto espaol. ( Grande )


viaje de regreso. ( Bueno )

noticia para daros. ( Malo )


de sus amigas les gusta el tango. ( Ninguno )

d) Quin ser el

alumno a sacar un sobresaliente este ao? ( Primero )

e) Dnde puedo encontrar


f) Le gusta ver el
g)

quiosco abierto ahora? ( Alguno )


hermano por la tele? ( Grande )

rpido puedes limpiar tu habitacin? ( Cuanto )

h) Creo que esta es una

solucin para este problema. ( Bueno )

I) La situacin del pas es

difcil actualmente. ( Tanto )

j) Silvia fue mi
l) Tengo

novia. ( Primero )
de estampillas repetidas en mi coleccin. ( Ciento )

ll) El lunes fue

da estupendo, hicimos muchas cosas juntos. ( Uno )

m) Julia esta con una


n)

cara y un

humor hoy. ( Malo - Malo )

persona puede conseguir un

empleo. ( Cualquiera - bueno )

ACENTUACIN
O acento a maior fora de uma determinada slaba ao pronunciar uma palavra. Ele
pode ocorrer na ltima slaba (aguda), na penltima (grave), na antepenltima
(esdrjula) ou na anterior antepenltima (sobresdrjula). Na lngua espanhola s
existe um acento grfico (la tilde) que se coloca sobre a vogal da slaba tnica.
Oxtonas (agudas)
So acentuadas as palavras terminadas em vogal, n ou s.
Exemplos: len, caf, sof, domin, quizs,...
Paroxtonas (graves)
So acentuadas as palavras terminadas em consoante, EXCETO n e s.
Exemplos: rbol, trax,...
Proparoxtonas (esdrjulas)
Todas so acentuadas.
Exemplos: oxgeno, anlisis, ejrcito,...
"Super proparoxtonas" (sobresdrjulas)

Todas so acentuadas.
Exemplos: explquemela, cmpratelo,...
Obs.: so verbos no imperativo + pronome.
Regra do Hiato (Regla del Hiato)
Esta regra tem por ordem acentuar as palavras que formam um hiato, que a separao
de uma vogal forte e de uma vogal fraca. Neste caso, colocamos um acento na vogal
fraca para assinalar a slaba tnica da palavra.
Vogais fortes: A, E e O.
Vogais fracas: I e U.
Exemplos: todav a (ainda), d a, fr o, analog a, tecnolog a, ba , gar a (garoa), pa
s, gr a (guincho), ma z (milho), contin a,...
TILDE DIACRTICA
La tilde diacrtica sirve para diferenciar palabras que se escriben de la misma forma
pero tienen significados diferentes. Es decir, es la que permite distinguir palabras con
idntica forma, escritas con las mismas letras, pero que pertenecen a categoras
gramaticales diferentes.
Con tilde o
acento diacrtico

Sin tilde o acento diacrtico


- Adverbio (cuando equivale a hasta,
tambin, inclusive o siquiera, con
negacin). Ejemplo: aun los sordos
aun
habrs de orme.
- Locucin conjuntiva. Ejemplo: aun
cuando.

de

Preposicin. Ejemplos: un vestido de


seda / Iros de aqu.

el Artculo: el soldado ya ha llegado.

Adverbio de
tiempo sustituible
por todava.
an
Ejemplos: an es
joven / No ha
llegado an.
Del verbo dar.
Ejemplos: d
d usted las gracias
/ Quiero que me
d este regalo.

Pronombre
personal.
Ejemplos: me lo
dijo l/ l no
quiere dar su
brazo a torcer.

Conjuncin adversativa. Ejemplos:


Adverbio de
mas quiso convencerlo, mas fue imposible / ms cantidad.
Lo saba, mas no nos quiso decir nada.
Ejemplos: hablas

ms, despacio /
Dos ms cinco
son siete.
- Posesivo. Ejemplo: Te invito a mi
casa.
mi
- Sustantivo como "nota musical".
Ejemplo: el mi ha sonado fatal.

se

Pronombre personal: se comi todo el


cocido.

Conjuncin condicional. Ejemplo : Si


llueve no saldremos / Todava no s si
si ir.
Sustantivo como "nota musical".
Ejemplo: una composicin en si bemol.

te

Pronombre Personal. Ejemplos: te lo


regalo / Te he comprado unos guantes.

tu Posesivo. Ejemplo: dame tu abrigo.

Pronombre
personal.
Ejemplos: a m
m
me gusta el ftbol
/ Tienes algo
para m?

Forma del verbo


ser o saber: yo no
s nada / S
buenos con ellos,
por favor.

Adverbio de
afirmacin o
pronombre
personal
reflexivo.
Ejemplos: s,
quiero! / Solo
habla de s
mismo.

Sustantivo
(bebida).
Ejemplo: toma
una taza de t.

Pronombre
personal.
Ejemplo: t
siempre dices la
verdad.

1) Classifique as palavras abaixo em: agudas, llanas, esdrjulas e sobresdrjulas


a) Camin
b) Ocano
c) Corrgemelo
ch) Perfeccin
d) Ftbol
e) Fcil

f) rboles
g) Cancin
h) Vndeselo
2) Coloque os acentos grficos corretamente nas palavras abaixo:
a) Medicacion
b) Caracter
c) Futbol
ch) Lagrima
d) Leon
e) Compralo
f) Papa
3) Seala la alternativa correcta:
1. Elige la respuesta correcta:
a)

Me gusta mucho ms visitar la Feria de Sevilla en el mes de Abril.

b)

Me gusta mucho mas visitar la Feria de Sevilla en el mes de abril.

c)

Me gusta mucho ms visitar la Feria de Sevilla en el mes de abril.

2. Elige la respuesta correcta:


a)

El problema para mi no son las crticas acerbas que me han dirigido.

b)

El problema para m no son las crticas acerbas que me han dirigido.

c)

El problema para m no son las crticas acervas que me han dirigido.

3. Elige la respuesta correcta:


a)

Si se dice s a la primera opcin mrcala con una cruz.

b)

Si se dice si a la primera opcin mrcala con una cruz.

c)

Si se dice s a la primera opcin marcala con una cruz.

4. Elige la respuesta correcta:


a)

T has sido elegida como actriz del ao por una revista estadounidense.

b)

Tu has sido elegida como actrz del ao por una revista estadounidense.

c)

Tu has sido elegida como actris del ao por una revista estadounidense.

5. Elige la respuesta correcta:


a)

El amigo de las palabras es una publicacin dedicada a la prosa potica.

b)

l amigo de las palabras es una publicacin dedicada a la prosa potica.

c)

El amigo de las palabras es una publicacin dedicada a la prosa poetica.

6. Elige la respuesta correcta:


a)

l historial de Hernesto era impresionante.

b)

El historial de Ernesto era impresionante.

c)

El istorial de Ernesto era impresionante.

7. Elige la respuesta correcta:


a)

El t verde contiene compuestos antioxidantes beneficiosos.

b)

El te verde contiene compuestos antioxidantes beneficiosos.

c)

El t verde contiene compuestos antiosidantes beneficiosos.

8. Elige la respuesta correcta:


a)

T he comprado una corbata en una tienda de moda.

b)

Te he comprado una corvata en una tienda de moda.

c)

Te he comprado una corbata en una tienda de moda.

9. Elige la respuesta correcta:


a)

Espero que el sueldo de para hacerle un regalo a mi abuela.

b)

Espero que el sueldo d para hacerle un regalo a mi abuela.

c)

Espero que el sueldo d para hacerle un regalo a m abuela.

10. Elige la respuesta correcta:


a)

Yo no s nada del alboroto en el saln de te.

b)

Yo no se nada del alboroto en el saln de t.

c)

Yo no s nada del alboroto en el saln de t.

11. Elige la respuesta correcta:


a)

Esta vez s lo haban detenido con pruebas culpatorias.

b)

Esta vez si lo haban detenido con pruebas culpatorias.

c)

Esta vez s lo habian detenido con pruebas culpatorias.

12. Elige la respuesta correcta:


a)

Voy a la playa, ms no podr acompaarte en el fueraborda.

b)

Voy a la playa, mas no podr acompaarte en el fueraborda.

c)

Voy a la playa, ms no podre acompaarte en el fueraborda.

13. Elige la respuesta correcta:


a)

A pesar de la huelga, el herniando sufri la operacin de hernia.

b)

A pesar de la uelga, el herniando sufri la operacin de hernia.

c)

A pesar d la huelga, el herniando sufri la operacin de ernia.

14. Elige la respuesta correcta:


a)

La hermana de Holga s comi una tarta.

b)

La hermana de Olga s comi una tarta.

c)

La hermana de Olga se comi una tarta.

15. Elige la respuesta correcta:


a)

La frase "slo se que no s nada", es atribuida a Scrates.

b)

La frase "slo s que no se nada", es atribuida a Scrates.

c)

La frase "solo s que no s nada", es atribuida a Scrates.

16. Elige la respuesta correcta:


a)

Pese a quien pese, el no quiere dar su brazo a torcer.

b)

Pese a quien pese, l no quiere dar su brazo a torcer.

c)

Pese a quien pese, el no quiere dar su braso a torcer.

17. Elige la respuesta correcta:


a)

A m lo que realmente me interesa es el arte, a l no lo s.

b)

A mi lo que realmente me interesa es el arte, a l no lo s.

c)

A m lo que realmente me interesa es el arte, a el no lo s.

18. Elige la respuesta correcta:


a)

El Supremo de Pakistn critica que aun no hay arrestos por el atentado.

b)

El Supremo de Pakistan critica que an no hay arrestos por el atentado.

c)

El Supremo de Pakistn critica que an no hay arrestos por el atentado.

19. Elige la respuesta correcta:


a)

El programa mas escuchado en la radio llega a su novena temporada.

b)

El programa ms escuchado en la radio llega a su novena temporada.

c)

El programa ms escuchado en la radio llega a su novena temporda.

20. Elige la respuesta correcta:


a)

El ermano de Ernesto es muy holgazn.

b)

El hermano de Hernesto es muy holgazn.

c)

El hermano de Ernesto es muy holgazn.

TEXTOS DE VESTIBULARES, ENEM E PAS


(MACK-SP) Texto para las preguntas de 1 a 4
El individualismo
El individualista no pretende vivir aislado; nadie puede ocuparse bien de s
mismo cuando se despreocupa de todo lo dems que le rodea.
Recientemente 1200 jvenes viajaron a Estrasburgo en un tren de la solidaridad
haciendo parte de la campaa organizada por el Consejo de Europa contra el racismo y
la intolerancia. Se recogieron all las declaraciones de un joven nacido en Martinica y
que vive en Pars, dedicado al dibujo de historietas. Cont episodios de rechazos a causa
del color de su piel sufridos en la gran urbe, una odiosa vergenza que por desgracia
tambin se conoce en Espaa. Durante el viaje a Estrasburgo dicho joven se siente bien
integrado aunque, dice, todava me cuesta aceptar ese individualismo de los pases
europeos que, a veces, te hace sentirte muy solo pese a tener buenos amigos. Las
sociedades individualistas modernas, probablemente, son menos clidas y acogedoras
que otras comunidades tradicionales donde cada cual se siente formando parte de un
gran todo. No es precisamente la mentalidad individualista la que admite o rechaza a los
seres humanos por el bloque racial al que pertenecen.
Esos jvenes que viajaron a Estrasburgo son individualistas en el verdadero
sentido de la palabra, al oponerse al racismo y a la intolerancia, porque no solamente se
preocupan con sus semejantes sino porque no conocen nada ms digno de aprecio que
cada uno de los individuos concretos.
El individualismo no ignora que cada ser humano es fruto de la colectividad en
la que nace y de la historia que comparte con otro; pero asegura que lo importante no es
lo que las circunstancias no elegidas hacen de nosotros sino lo que nosotros, eligiendo,
hacemos con esas circunstancias.
Fernando Savater. In: El Pas (adaptado).

1. Segn el texto podemos afirmar que el individualismo en los pases europeos te hace
sentir:
a) solo aunque tengas buenos amigos.
b) integrado a la comunidad de amigos.
c) amarrete en la comunidad de amigos.
d) un ser humano derrochador en el crculo de amistades.
e) cercado de amigos.
2. El texto define el individualismo como:
a) resultado de ausencia de prejuicios.
b) resultado de la colectividad en la que nace y de la historia que comparte con la
sociedad a la que pertenece.
c) resultado de la tolerancia de una sociedad para con el ser humano.
d) no predominante en las grandes urbes.
e) armoniosa convivencia, en colectividad, dando origen a generosos seres humanos.
3. En el texto, podemos afirmar que:
a) al despreocuparse de los dems, el individualista consigue vivir aislado.
b) el individualista anhela vivir aislado y despreocuparse de los dems.
c) el individualista no aspira a vivir aislado, pues no conseguir despreocuparse de los
dems.
d) al despreocuparse de los dems obtiene el aislamiento de la sociedad que le es
necesaria para lograr vivir solo.
e) en la gran urbe difcilmente el individuo es rechazado por el tono de su piel.
4. a interpretacin correcta de la frase destacada en el texto, ...te hace sentirte muy solo
pese a tener buenos amigos, es:
a) sentirse aislado en la multitud.
b) sentirse querido por la comunidad.
c) sentir que a su alrededor reina la amistad.
d) sentir apego al afecto de la comunidad.
e) sentirse integrado y acogido por la comunidad.
Texto para las preguntas de 5 a 7
Cortesa y educacin: las buenas costumbres
Durante siglos ha sido norma de cortesa que los hombres cedieran el paso a las mujeres
y los jvenes lo cediesen a los mayores. Claro est que las actuales condiciones de vida
en las ciudades (transportes urbanos abarrotados, colas de supermercados en las que la
gente intenta avanzar una posicin desesperadamente, caravanas de vehculos en las
carreteras etc.) han hecho que la norma se haya relajado enormemente.
Tambin sera de esperar que quien cruza delante de nosotros una puerta, tuviese
la amabilidad de sostenerla para que passemos y no la dejase caer en nuestras narices.
Pero, en fin, los tiempos son otros y si usted sigue empeado en ser amable e ir
cediendo el paso y sosteniendo puertas, pronto descubrir que muchsima gente, en su
ajetreo inexplicable, ni se molestar en darle las gracias.
Angel Amable. Manual de buenas costumbres (adaptado).
5. En el texto, podemos afirmar que:

a) el comportamiento de los hombres era, hasta tiempos atrs, engorroso.


b) los jvenes no solan cederles el asiento a los mayores, tiempos atrs.
c) es correcto afirmar que las condiciones de vida de antiguamente, no permitan que el
hombre tuviese amabilidad en sus actos.
d) los tiempos actuales se comparan igualmente con los siglos anteriores, manteniendo
las normas de cortesa.
e) antiguamente el hombre era un caballero y se preocupaba en complacer a una dama.
6. En el texto, ngel Amable afirma que las causas del cambio de comportamiento
humano deben achacarse a:
a) los anhelos de nuestros abuelos.
b) a la falta de preocupaciones de la situacin actual.
c) las vicisitudes de la vida moderna.
d) a la influencia del entorno de nuestros antepasados.
e) a la proliferacin de las termitas.
7. El autor es terminante al afirmar que:
a) los retoos son responsables por los cambios en las formas de tratamiento.
b) es precipitado atribuirle la culpa al presunto hombre moderno.
c) hubo un cambio brusco en las actitudes cotidianas del hombre moderno.
d) es pasajero este comportamiento brusco y grosero.
e) difcilmente el hombre moderno optimizar sus actitudes en relacin a su prjimo.
Texto para las preguntas de 8 a 10
El Spanglish
El spanglish avanza, (nos guste o no nos guste). Es una realidad. La mezcla del
espaol y del ingls se ha convertido en una especie de lengua que se utiliza cada vez
con ms frecuencia en las regiones estadounidenses donde abunda la poblacin hispana.
Es un destrozo de ambos idiomas, por supuesto. Pero hay mucha gente que ya no
sabra hacerse entender solamente con uno, y la constante mezcla del ingls y del
espaol como algo absolutamente lgico, se ha convertido en su habitual forma de
expresin.
Natalia Figueroa. El spanglish. In: ABC (adaptado).
8. De acuerdo con la autora, podemos afirmar que:
a) el spanglish es rechazado como tercera lengua por los estadounidenses.
b) a pesar del destrozo hay mucha gente que sabe hacerse entender con una sola lengua.
c) donde abunda la poblacin hispana el dominio de una sola lengua es evidente.
d) nos guste o no nos guste esta juncin de lenguas tiene sus das contados.
e) el spanglish estropea dos lenguas: el espaol y el ingls.
9. La expresin destacada en el texto, por supuesto, significa:
a) Insistentemente.
b) Raramente.
c) Lgicamente.
d) A menudo.
e) A lo mejor.
10. La expresin destacada en negrita, nos guste o no nos guste, est conjugada en:

a) Presente de subjuntivo.
b) Imperfecto de subjuntivo.
c) Pretrito pluscuamperfecto de subjuntivo.
d) Imperativo negativo.
e) Pretrito indefinido de indicativo.
(UEL- PR) Texto para las preguntas 11 y 12
La educacin primaria bilinge, tanto para nios en edad escolar como para
indgenas adultos, alcanz un notable auge en la dcada de los aos cincuenta en pases
como Mxico y Per.
La mera educacin bilinge implica la existencia de dos lenguas como vehculo
de enseanza en la educacin preescolar y primaria y su objetivo parece ser la
adquisicin del bilingismo mediante el paulatino cultivo de una segunda lengua,
utilizando ambas lenguas en el desarrollo de los programas de enseanza. Es lo que
ocurre en los centros de lite hispano-alemanes, anglohispanos o hispanofranceses
existentes en muchos pases iberoamericanos y que no supone la formacin en otra
cultura distinta, salvo en aquellos modos culturales que pudieran asimilarse de manera
indirecta, sin ninguna intervencin, al
propio tiempo que se adquiere la segunda lengua, ya que es bien sabido que un idioma
no es solamente un medio de comunicarse sino una forma concreta de expresar un modo
de pensar y de ser.
Sin embargo, esta educacin bilinge no alcanza a ser una formacin bicultural,
entendiendo por tal la capacitacin en todo un conjunto de modos de vida, formas de
proceder y de pensar, costumbres, gustos y comportamientos y la asimilacin de una
serie de valores enraizados con esta forma caracterstica de comportarse.
Biculturalismo es otra cuestin. La educacin bilinge bicultural es aquella que
pretende proporcionar una formacin y una capacitacin en dos culturas a travs de dos
lenguas, sobre la base de un absoluto respeto hacia ambas. Algunas de las experiencias
y proyectos educativos llevados a cabo en los aos sesenta en los pases
iberoamericanos bajo la denominacin de biculturales no pasaron, sin embargo, de
una mera educacin bilinge, debido sobre todo a la falta de maestros debidamente
preparados.
Ocurre que en los pases iberoamericanos se confunden y se tienen como
sinnimos los trminos bicultural e intercultural, si bien ste ltimo se abri camino
como una nueva denominacin que lograse desterrar la confusin producida por el
empleo del trmino bicultural en proyectos educativos que no pasaron de ser de
educacin bilinge.
No obstante, el trmino intercultural tiene tambin un matiz relativo a la
democratizacin de la educacin, ya que los proyectos iniciados bajo esta denominacin
intentan anular la discriminacin lingstico-cultural producida por la colonizacin,
propugnando una articulacin pluralista de las diferentes etnias. El trmino
intercultural, por otra parte, es ms ajustado a la realidad de los pases
iberoamericanos ya que con esta expresin se espera respetar ms el pluralismo cultural
de muchos de ellos.
Disponvel em <http://www.campus-oei.org/oinvirt/rie04a05.htm>.
Acesso: 10.11.2003. (Adaptado)
11. De acordo com o texto:

a) na educao bilnge, o aluno formado na segunda cultura ao mesmo tempo que


adquire uma segunda lngua.
b) os alunos da educao bilnge s recebem uma capacitao especfica na segunda
cultura, desvinculada da formao lingstica.
c) os termos bicultural e bilinge so sinnimos e caracterizam adequadamente a
maioria dos projetos educativos desenvolvidos nos pases ibero-americanos.
d) a noo de biculturalismo implica em capacitao e formao em uma segunda
lngua e cultura.
e) com a educao bilnge intercultural pretende-se eliminar a diferena lingstica
existente entre vrias etnias indgenas.
12. Observe as afirmaes a seguir.
I. A educao bilnge bicultural se prope a anular a relao discriminatria imposta
pelos colonizadores
aos nativos americanos.
II. Na experincia ibero-americana houve projetos de educao meramente bilnge que
se denominavam projetos de educao bicultural.
III. Teoricamente, no h diferena entre educao bilnge e educao bilnge
bicultural, pois aprender uma lngua supe conhecer tambm o modo de pensar e de ser
do outro.
IV. A educao bilnge bicultural entende que a cultura um complemento
capacitao lingstica.
Esto corretas apenas as afirmativas:
a) I e II.
b) I e III.
c) II e IV.
d) II, III, IV.
e) I, III e IV.
(UNIVATES) Texto para las preguntas 13 a 17
El estrs en el trabajo
El estrs es una respuesta del organismo ante la presencia de situaciones que
implican cambio; muchas personas pueden adaptarse a estas circunstancias, pero otras
responden de manera inadecuada. Los expertos en el tema revelan que el estar muy
estresado de manera continua puede producir problemas serios en la salud, como
dolores de cabeza, aumento de la presin arterial y enfermedades cardiovasculares.
Hoy en da el estrs hace parte de la vida diaria de las personas, en especial en el
mbito laboral y en los horarios de llegada al trabajo. Por tal motivo, es importante
conocer las manifestaciones clnicas del estrs laboral para poder instaurar a tiempo
medidas que controlen estas respuestas al cambio.
La sensacin de no tener control sobre lo que est pasando es una fuente comn
de estrs laboral. Es bueno revisar el manual del cargo de manera que se tenga claro las
responsabilidades y lmites. El temor a perder el empleo es una constante hoy en da,
pero permitir que la tensin por esta causa disminuya la efectividad y calidad en el
trabajo, peora el cuadro de estrs. Lo mejor es dirigir las energas a relajarse y a cambiar
la programacin mental hacia una actitud positiva.
El estrs laboral hace parte de la vida y es necesario saber enfrentarse ante los
cambios continuos que ocurren da a da. No se puede, ni se debe, evitar los problemas;

cuando hay la decisin de hacerse cargo de una situacin, se tiene ya ms de la mitad


del camino ganado.
Salud Hoy. Dic. 2000.
13. La idea central del texto es:
a) el mundo pasa por una crisis econmica actualmente.
b) todas las personas son estresadas.
c) el estrs es el nico causador del aumento de la presin arterial.
d) el estrs est presente diariamente en la vida de las personas.
e) evitar los problemas es la solucin para el estrs.
14. En el texto est escrito: ... presin arterial y enfermedades cardiovasculares.... De
acuerdo con las reglas de uso de eufona (y o e) se puede decir que la nica alternativa
correcta es:
a) hija e hijo.
b) geografa y historia.
c) ratn e elefante.
d) verano y invierno.
e) ella e yo.
15. De las frases abajo, la nica que emplea correctamente el artculo neutro lo es:
a) Lo muchacho rubio es muy guapo.
b) Mara habl acerca de lo libro de literatura en la clase.
c) Los turistas ahora saben lo bonitas que son nuestras playas.
d) Ayer fue lo da ms importante de mi vida.
e) No puedo hacerlo ahora.
16. La expresin hacia en el texto nos expresa idea de:
a) posesividad.
b) distancia.
c) aproximacin.
d) cambio.
e) direccin.
17. Leyendo el texto, observa las afirmaciones siguientes.
I. Las personas sienten miedo de perder el empleo hoy en da.
II. Es importante saber las causas del estrs para poder controlarlo.
III. Los horarios de llegada al trabajo son de menos estrs.
a) Est correcta la alternativa I.
b) Estn correctas las alternativas I y II.
c) Estn correctas las alternativas II y III.
d) Est correcta la alternativa III.
e) Todas las alternativas estn correctas.
(UNIVATES) Texto para las preguntas 18 a 20
A correr
Las calles mostraron sus mejores momentos de entusiasmo. La carrera ya
comenz! Dej escucharse el grito. Era el maratn. Una competencia sin lo clsico: ni la
salida ni la llegada en interior de un estadio. Arrancaron cientos y cientos de hombres y

mujeres. Comenz en la parte norte del poblado, por el acceso de donde se llega de
Tijuana. Eso para ser exactos. Todos en la calle a lo largo de la avenida Jurez, en
carretera hasta llegar a Puerto Nuevo.
Todo cubre una distancia de medio maratn dicen que son 22 kilmetros y
que lo tradicional es evocar la recorrida por un soldado para anunciar a los atenienses la
victoria de los griegos sobre los persas.
El Maratn de Rosarito. Era el reto a la resistencia a la fatiga, casi diramos
inagotable. Muchos al trote cogiendo senderos por pavimentos variados que parecan
difciles aun para una simple marcha.
De grupos de hombres y mujeres, de visitantes, turistas y locales. De todas las edades.
Venan de Los ngeles, de otros condados. Haba de Tijuana, Tecate, Mexicali,
Ensenada y claro de Rosarito!, expresin genuina de los locales. Era el momento de
correr sin esa arrogancia majestuosa de los grandes maratonistas. Baados de sudor los
torsos, unos musculosos, otros desfigurados al estar pasados en kilos.
De observar aquellos rostros redondos o afilados, as como suena, clasificacin
sin precedentes a cargo de la vox populi, eso para diferenciar entre aquel gordo o ese
flacucho que como competidores genuinos se mezclan para demostrar que no hay
fijn!.
Caras o como dice el gringo de face juvenil, bien medio maduros y muy
maduros y an aquellas que dibujan los inevitables rasgos de la llamada tercera edad.
Fue el destacar aquella sensibilidad del esfuerzo realizado y que perlaba la frente
y los pmulos. Como si todo eso permitiera sostener aquella figura imprescindible en
esa carrera buscando llegar a la meta.
Porque todos los competidores pertenecan a un cuadro que conforme al
desarrollo de la carrera marcaba la oportunidad de la convivencia.
Era el deseo de conquistar la meta! El lugar? Eso no import! El nico inters
era la meta, de haber llegado, de haber vencidos esos kilmetros dejados a tras como
constancia de que si se pudo! Pareca que toda la poblacin, toda ella tomaba parte en
la fiesta deportiva. Unos como espectadores, otros como actores en el regocijo.
Nada mejor que correr siguiendo el itinerario como si fuera un desfile.
El Mexicano, 11 oct. 2004.
18. El texto aborda uno de los temas abajo. Cul es?
a) Una famosa feria de Mxico.
b) Una caminata pidiendo paz.
c) El maratn de Tijuana.
d) La Semana Santa de Mxico.
e) El maratn de los actores mexicanos.
19. La palabra an, puede ser sustituida, sin alterar el sentido contextual, por:
a) pero.
b) aunque.
c) sin embargo.
d) todava.
e) mientras.
20. La expresin a lo largo de la puede ser traducida al portugus por:
a) a largura da.
b) ao longo da.
c) longe da.

d) a margem da.
e) perto de.
Texto para las preguntas 21 y 22
Crisis en Castilla y Len
El Instituto de Poltica Familiar de Castilla y Len alerta sobre la situacin matrimonial
de las parejas espaolas, en especial de la regin, ya que 4.887 han roto en el ltimo
ao. Es decir, cada da se han producido una media de 13 rupturas.
El pasado 17 de septiembre, el Gobierno aprob el anteproyecto de ley para la
reforma del divorcio. Segn el Instituto de Poltica Familiar esta decisin la ha tomado
dando la espalda a las asociaciones familiares, algo que Amparo Valcarcel, secretaria de
Estado, haba prometido. Eduardo
Hetfelder, presidente de la asociacin, seala que esto es un gran error poltico ya que
se ha dado el primer paso para aprobar una ley de una manera unilateral y sin contar con
los expertos.
La situacin de ruptura familiar preocupa a la asociacin que cree urgente
abordar las causas de estas rupturas as como aportar soluciones encaminadas a
reducirlas. La principal medida que plantean es crear una poltica integral de familia de
carcter universal y no asistencial. Adems, exigen la creacin de la Secretara de
Estado de Polticas Familiares con capacidad organizativa y presupuestaria para
desarrollar la promocin y proteccin del ncleo familiar.
Tambin creen necesaria la universalizacin de las prestaciones sociales por hijo
a cargo y su equiparacin a la media europea, 2,2% del PIB; as como la
universalizacin de la paga de 100 euros para hijos menores de tres aos, pudiendo
solicitarla cualquiera de los padres, sea o no trabajador.
Otras medidas que contemplan como necesarias son elaborar un pacto entre la
administracin central y los ayuntamientos para abaratar la vivienda, la reforma de la
ley del IRPF con perspectiva de la familia, el apoyo a la maternidad, el impulso de
medidas preventivas para ayudar a superar las crisis matrimoniales y la puesta en
marcha del Observatorio de la Familia.
El Adelanto de Salamanca. 11 oct. 2004.
21. El verbo han roto puede ser traducido al portugus, sin cambiar su sentido, por:
a) romperam.
b) tm quebrado.
c) haviam rompido.
d) tm rodado.
e) voltaram.
22. La idea central del texto es:
a) los problemas polticos de Castilla y Len.
b) leyes para hijos de parejas espaolas separadas.
c) crisis econmica en Castilla y Len.
d) la necesidad de abaratar la vivienda en esta regin.
e) la preocupacin con las rupturas matrimoniales en Espaa.
(UEL PR) Texto para las preguntas de 23 a 25
El miedo an viaja en los trenes

El ambiente que se respira en el tren de la lnea C-7 entre Prncipe Po y Alcal


de Henares es tenso. Unos se sumergen en la lectura del peridico, otros miran
fijamente por la ventana o simplemente al suelo. Nadie habla. Hay quienes no dejan de
mirar una y otra vez el reloj. Cundo llegaremos por fin?, parecen pensar.
Pero desde el 11 de marzo ya nada es como antes en la red de cercanas de
Madrid. Y a los cerca de un milln de personas que utilizan este servicio a diario el
trauma los perseguir todava por mucho tiempo. Un gesto, algn pequeo detalle,
muchas veces son suficientes para que los ojos se llenen de lgrimas. Como el crespn
negro que el seor de enfrente lleva en seal de luto. O como un anuncio de la empresa
estatal de trenes Renfe que publicaban hoy los diarios: Gracias por vuestra ayuda, por
vuestras lgrimas, por vuestra solidaridad, por estar a nuestro lado y al lado de todas las
vctimas del horror. Gracias a todos de corazn.
Mientras el tren contina su camino, en el mismo peridico la mirada se fija en
la lista de los 182 muertos ya identificados.
La mayor, Alicia, tena 63 aos. La menor se llamaba Patricia, era de Polonia y
apenas tena siete meses. Aunque eso no es del todo correcto: en el listado, donde pone
edad, aparece tambin la palabra feto. Era un beb que nunca lleg a nacer. A
Anabel, su madre, la muerte la sorprendi con 29 aos.
Disponvel em <http://www.iblnews.com/noticias/03/103419.html>. Acesso: 17 mar.
2004. (Adaptado)
23. Sobre o texto correto afirmar que:
a) as vtimas do atentado de 11 de maro no sero mais lembradas pela populao de
Madri.
b) a populao de Madri no ouvir mais as ameaas dos terroristas responsveis pelo
atentado de 11 de maro.
c) a populao de Madri deseja esquecer a ao terrorista ocorrida em 11 de maro.
d) as vtimas do atentado de 11 de maro em Madri no sero esquecidas.
e) a cidade de Madri no apoiar mais as aes de grupos terroristas espanhis.
24. Pero desde el 11 de marzo ya nada es como antes en la red de cercanas de
Madrid. Com base neste fragmento do texto, correto afirmar que:
a) desde 11 de maro, o servio de transporte ferrovirio de Madri e regio est
trabalhando com sua capacidade normal.
b) com o atentado de 11 de maro, foram cancelados os servios de transporte
ferrovirio de Madri e regio.
c) desde 11 de maro, a tenso tomou conta dos usurios do servio de transporte
ferrovirio de Madri e regio.
d) desde 11 de maro, o servio de transporte ferrovirio deixou de atender as
localidades vizinhas a Madri.
e) desde o atentado de 11 de maro, os usurios deixaram de utilizar os servios de
transporte ferrovirio de Madri e regio.
25. A empresa estadual de trens Renfe publicou nos jornais dirios:
a) uma orao dirigida s pessoas que mostraram solidariedade s vtimas do atentado.
b) um anncio contendo uma lista de 182 desaparecidos desde a data do atentado.
c) um anncio pedindo oraes dirias pelas vtimas do atentado de 11 de maro.
d) uma orao em homenagem s vtimas Alicia, de 63 anos, e Patricia, de sete meses.
e) um anncio em agradecimento a todos pela ajuda e solidariedade para com as vtimas
do atentado.

Texto para las preguntas 26 y 27


Brasil y sus fronteras
LA IMPORTANCIA DE LA PRESENCIA HISPNICA EN BRASIL, TANTO
DESDE UNA PERSPECTIVA HISTRICA COMO EN LA SITUACIN ACTUAL
La historia nos lleva a remontarnos a los viajes exploratorios de Cristbal Coln
por la costa de Sudamrica, a la decisiva importancia del Tratado de Tordesillas y a la
unin con Felipe II de las coronas de Espaa y de Portugal. Los acontecimientos ms
recientes nos obligan a recordar la notable incidencia de la inmigracin espaola
durante los ltimos cien aos, que tuvo como territorios de destino principales el sur y
el sudeste de Brasil, lo que, unido a la vecindad de los pases hispnicos, contribuy a
dar al espaol una presencia relativamente apreciable y a convertirlo, en el sur, en una
lengua cercana
y familiar en todo tipo de relaciones. En los inicios de un nuevo siglo, el espaol en
Brasil es una lengua de prestigio y en expansin, puesto que es vista como una lengua
internacional de indudable utilidad debido al nacimiento y a la consolidacin del
Mercado Comn del Sur (Mercosur), a la magnitud de la presencia econmica de
Espaa y a la influencia de la cultura hispnica en su conjunto.
El contacto histrico de Brasil con los vecinos pases hispanohablantes tiene su
muestra ms clara en las relaciones personales y comerciales establecidas en la frontera
con Paraguay, Argentina y Uruguay, que han llegado a transformarse en el Mercosur.
Pero ni el espaol ni la presencia hispnica en Brasil terminan en los territorios de la
frontera surea, por muy importantes que sean, y realmente lo son. Adems de con sus
vecinos y socios poltico-econmicos, Brasil tiene frontera con Bolivia, Per, Colombia
y Venezuela, y mantiene buenas relaciones con Ecuador y Chile, nicos pases
hispanohablantes sudamericanos con quien Brasil no limita.
MORENO FERNNDEZ, Francisco. El espaol en Brasil: El espaol en el mundo. In:
Anuario del Instituto Cervantes. Madrid: Instituto Cervantes, 2000. p.197-228.
26. Com base no texto correto afirmar que:
a) o Tratado de Tordesilhas e a unio das coroas de Espanha e Portugal converteram o
espanhol em uma lngua essencial para o desenvolvimento econmico do Brasil.
b) no Brasil, o espanhol adquiriu prestgio e se expandiu devido a vrios fatores, entre
eles o nascimento e a consolidao do Mercosul.
c) a histria do Brasil, sua colonizao e a fronteira com os pases que falam o espanhol
trouxeram a necessidade da criao de um mercado comum entre estes pases e a
Espanha.
d) a grande imigrao espanhola para territrios brasileiros s foi possvel por meio da
unio das coroas de Espanha e Portugal.
e) o Brasil mantm relaes scio-polticas somente com os pases que falam o espanhol
e com os quais faz fronteira.
27. Com base no texto, considere as afirmativas a seguir.
I. A valorizao da lngua e da cultura espanhola no Brasil s foi possvel devido ao
acordo econmico e comercial entre os pases da Amrica e da Espanha.
II. O espanhol utilizado nas regies Sul e Sudeste do Brasil fomentou a comunicao
nas transaes comerciais.

III. A imigrao espanhola nas regies Sul e Sudeste do Brasil e a proximidade dos
pases hispnicos contriburam para que a lngua espanhola, nos ltimos anos, estivesse
presente em todas as relaes.
IV. O ensino da lngua espanhola no Brasil se restringe aos limites geogrficos das
regies mais desenvolvidas do pas.
Esto corretas apenas as afirmativas:
a) I e IV. d) I, II e III.
b) II e III. e) I, II e IV.
c) III e IV.
Texto para la pregunta 28
Samba en la favela
Kelly lleva el ritmo en el cuerpo. A sus siete aos, esta mulata es la benjamina
del grupo Lactogirls. Son las 11 de la maana de un martes y Jaciara, alias Negona
(Negrona) la cantante del grupo, una chica de 23 aos robusta y sonriente , se deja
las cuerdas vocales ante el micrfono. Sus compaeras sudan mientras golpean los
tambores, timbales, bongs. La msica retumba en todo Candeal, la favela de Salvador
de Baha en la que viven y ensayan. Es el espritu de Candeal: con ganas se puede
conseguir mucho. El mismo espritu que enganch a Fernando Trueba. El director se
traslad con su equipo a Salvador de Baha para rodar en Candeal, una favela que en los
ltimos 20 aos ha sufrido una transformacin sorprendente. Nada ms pisarlo supe
que hara algo aqu, explica. Ese algo es una pelcula llena de msica, cuyo ttulo lo
dice todo: El milagro de Candeal. Trueba conoci la historia de este barrio de la mano
de su ciudadano ms ilustre, Carlinhos Brown. Cuando sus xitos musicales empezaron
a reportarle ingresos, el percusionista decidi mejorar las condiciones de vida de la
favela donde l mismo naci hace 42 aos.
Uno de cada cuatro de los 2.400.000 habitantes de Salvador de Baha vive en
una favela, un barrio de infraviviendas. Candeal es una de ellas, aunque sus habitantes
prefieren hablar de comunidad. Segn el nico censo realizado en Candeal y que data
de 1997, el 77% de los hogares subsiste mensualmente con un salario mnimo: 260
reales, unos 70 euros. Una miseria. Aunque eso es lo nico que Candeal tiene de favela:
no hay violencia, no se ven drogas, no se oyen tiros. Lo que s se puede escuchar es
msica, a todas horas y en cada esquina. El artfice de este milagro es Antnio Carlos
Santos de Freitas, conocido como Carlinhos Brown. l creci en Candeal cuando el
barrio no era ms que una selva de caminos de tierra.
El padre de Carlinhos se ganaba la vida pintando fachadas; su madre lavando ropa. A
los 10 aos, l tambin se puso a trabajar, llevando agua de casa en casa. As comenz a
forjarse su conciencia social. Senta mucha vergenza de la situacin que vivamos en
mi casa, hasta que empec a ver que era la misma que la de mis vecinos. No tena el
coraje de cobrarles el agua a las embarazadas, dice.
Carlinhos empez a tocar en los mismos baldes que usaba para repartir agua. A
los 12 aos conoci al Maestro Pintado del Bong, un percusionista que se ganaba la
vida como chfer del gobernador de Baha y que fue su primer maestro.
Durante un carnaval, Caetano Veloso escuch a Carlinhos al frente de los
timbales y lo incorpor a su banda. A la vuelta de una gira por Europa, Brown regres a
su barrio y fue testigo de una redada policial que acab con la vida de cinco de sus
habitantes. Entonces, algo se removi en su interior. Compr cien timbales y ense a
sus vecinos a tocar. As naci el grupo Timbalada, que hoy es uno de los ms famosos
de Brasil. El artista ha instalado en la favela un estudio de grabacin, al que acuden

msicos como Ricky Martin o Manolo Garca. Otra de las aportaciones de Carlinhos es
la Escuela Profesional de Msica Pracatum, en la que 150 nios y 170 adolescentes del
barrio estudian msica popular brasilea.
En 2002, Carlos Lpez, director de la discogrfica BMG, viaj a Salvador de
Baha para fichar a Carlinhos Brown. Cuando descubr cmo la msica poda cambiar
la espantosa realidad de una favela, pens que era una historia digna de ser contada en
una pelcula, recuerda. Tres das ms tarde coincidi con Fernando Trueba y le cont
lo que acababa de ver. El pasado enero comenz el rodaje de El milagro de Candeal,
donde se narra el reencuentro del msico Bebo Valds con sus races africanas y en la
que han participado Caetano Veloso, Gilberto Gil, Marisa Monte y el propio Carlinhos
Brown. La cinta se estrena a primeros de octubre. He querido hacer una pelcula que
retrate la belleza y el talento, no la pobreza, seala Trueba. Varios meses de rodaje han
hecho que el director se quede prendado de Candeal. Cuando pasea por la favela, llama
a cada nio por su nombre. Son ms guapos y felices que los nios de Madrid o Nueva
York, dice.
Disponvel em <www.marieclaire.es>. Acesso: 24 ago. 2004.
28. Com base no texto, correto afirmar que:
a) Carlinhos Browm aperfeioou sua formao musical na Escola Profissional de
Msica Pracatum.
b) trezentos e vinte alunos do Candeal estudam na Escola Profissional de Msica
Pracatum.
c) a Escola Profissional de Msica Pracatum abriga em suas instalaes crianas e
adolescentes de rua nascidas no Candeal.
d) Carlinhos Browm mantm um estudio de gravaes na Escola Profissional de Msica
Pracatum, onde gravam artistas famosos de renome nacional e internacional.
e) na Escola Profissional de Msica Pracatum h um estdio cinematogrfico fundado
pelo cantor Carlinhos Brown.
La irresistible seduccin del lado oscuro
Carmen Maana
Vampiros, hombres lobo, ngeles cados, zombis, fantasmas, hechiceros ...
siempre han habitado en las libreras. A veces silenciosos. A la espera. Ocupando unas
pocas baldas en un rincn de la tienda. Objeto de deseo exclusivo de adictos a la
fantasa ms oscura. Pero ha llegado su momento. Las criaturas del submundo han
salido a la luz y tomado los lugares privilegiados de las libreras y conquistando al gran
pblico. En el origen, este ltimo resurgimiento est en la saga Crepsculo, de
Stephenie Meyer, editada en Espaa en 2005. El vertiginoso fenmeno que desencaden
astronmicas cifras de ventas y beneficios, y la casi inagotable capacidad de explotacin
del producto puso los dientes largos a las editoriales. Si el pblico no quera muertos,
ellos les saciaran con un catlogo de monstruos.
Una oferta que se ampla al mezclar lo fantstico con otros gneros, de novela
ertica a negra, pasando por la reinterpretacin de los clsicos. Es una fiebre oscura que
enajena a editoriales y lectores. Y lo hace mediante amores imposibles, violencia, la
eterna lucha del bien contra el mal y la recreacin de unos mundos paralelos y
tenebrosos, siguiendo el ejemplo de lo que J. R. R. Tolkien hiciera en 1954 con El
Seor de los nillos. El primer gran fogonazo internacional de la temporada lo dar
Cornelia Funke con Carne de piedra, primera parte de su triloga Reckless. He aqu dos
elementos encontrados en este tipo de novela.
ngeles: Ni asexuados, ni inocentes. El ejrcito de criaturas celestiales que ha

desembarcado en las libreras no conoce la misericordia, pero s la pasin. Y, adems,


amenaza con arrebatar el trono de la fantasa a los chupasangres. La prueba es que Anne
Rice, creadora de la saga superventas Crnicas vampricas, ha cambiado los colmillos
por las alas en su ltima novela, La hora del ngel. Ediciones B publicar en Espaa el
15 de septiembre esta historia sobre la bsqueda de la salvacin, primera entrega de una
serie bautizada como Canciones de Seraphin. El resurgimiento de los querubines
comenz hace un ao con Angelology, de Danielle Trussoni. Will Smith ya ha
comprado los derechos para llevar al cine esta novela, traducida a 32 idiomas y
protagonizada por los nefilim, fruto de la unin entre un hombre y un ngel. La
triloga de Cassandra Clare, Cazadores de sombras, tambin se centra en estos seres tan
terriblemente bellos como crueles. Un poco ms edulcorados son los ngeles de
Oscuros, condenados a no poder estar nunca juntos. Y, ya diabticos, los de Halo, que
sale a la venta el 30 de agosto. Su escritora, Alexandra Adonetto, de tan solo 17 aos,
recrea un romance entre una ngel novata y el chico ms guapo del instituto.
Amor imposible: He cruzado ocanos de tiempo para encontrarte. Desde que
Drcula le susurrase esta frase a su amada en la obra homnima de Bram Stocker, qued
claro que ningn amor es tan profundo como el amor de ultratumba. Puede que los
protagonistas de estas novelas sean muertos vivientes o no tengan alma, pero se
enamoran: entre s o de humanos. Aunque siempre hay un obstculo que salvar. Como
en la tetraloga Medianoche, donde uno de los miembros de la pareja protagonista es un
vampiro y el otro un cazador de vampiros. El amor es tan o ms importante que la
fantasa dentro del argumento de estos ttulos. Por algo uno de los subgneros ms
importantes que componen este fenmeno se denomina romance paranormal.
Texto adaptado. Disponible en:
<http://www.elpais.com/articulo/portada/irresistible/seduccion/lado/oscuro/elpepuculba
b/20100828elpbabpor_15/Tes. 28/08/2010>.
Acceso el 03/09/2010.
Questo 16
Respecto a las obras de qu trata el texto, es correcto afirmar que
01) la novela El Seor de los Anillos ha sido la primera sobre el tema de la oscuridad.
02) en Oscuros los ngeles estn condenados a la soledad.
04) Halo es una novela de adolescente para adolescente.
08) La Hora del ngel trata de la busca de la salvacin.
16) Medianoche aborda el amor entre un vampiro y un humano.
Questo 17
Segn el texto, es correcto afirmar que
01) novelas sobre las criaturas del submundo slo les interesan a los adictos a la fantasa
ms oscura.
02) el xito de ese tipo de literatura ha conquistado el inters de las Editoriales.
04) para atender al pblico, se han sustituido en las historias los muertos por los
monstruos.
08) ese tipo de literatura ha tenido xito porque mezcla distintos gneros.
16) Will Smith llevar al cine la historia de La Hora del ngel.
Questo 18
De acuerdo con el texto, es correcto afirmar que

01) el verbo habitar (lneas 2-3) est usado en un sentido metafrico.


02) el verbo llegar (lnea 6) quiere decir que en la actualidad ese tipo de literatura
vende.
04) el verbo dar (lnea 28) expresa que Carne de Piedra de Cornelia Funke ganar
premios.
08) el verbo comenzar (lnea 45) se refiere a la reaparicin de los ngeles cuando
Trussoni publique su obra.
16) el verbo salir (lnea 55) indica que la obra empieza a ser vendida a partir de dicha
fecha.
Questo 19
Sobre los elementos que suelen conformar las obras, es correcto afirmar que
01) los ngeles se presentan como seres tan malos como extraordinariamente hermosos.
02) el amor es siempre imposible debido a la paranormalidad en los romances.
04) los nefilim son hijos de humanos y ngeles.
08) los muertos vivientes a menudo se apasionan por seres humanos.
16) lo ttrico y lo romntico se mezclan en tramas llenas de magia y sangre.
Questo 20
Respecto a los aspectos gramaticales y lxicos de la lengua espaola presentes en el
texto, es correcto afirmar que
01) primer (lnea 27), gran (lnea 27) y ningn (lnea 63) son palabras apocopadas
porque anteceden sustantivos masculinos singulares.
02) dos (lnea 30) es un numeral invariable en gnero.
04) lo (lnea 18) es un artculo determinado masculino singular.
08) el sustantivo ocanos (lnea 59) es un ejemplo de vocablo heterogenrico.
16) en la frase Puede que los protagonistas de estas novelas sean muertos vivientes ...
(lneas 64-65), la gramtica de la lengua espaola admite tanto la forma verbal sean
como son.
Los chicos se divierten cada vez ms puertas adentro
Privilegian la TV, la msica e Internet a las salidas. Segn una encuesta, son las
actividades que adolescentes y preadolescentes realizan con mayor frecuencia.
La influencia de los medios de comunicacin, junto al avance incesante de la
tecnologa, es total en los adolescentes argentinos. La actividad al aire libre va
quedando atrs, a medida que la cultura del entretenimiento se expande y coloniza los
hogares. En plena era de las pantallas, suena lgico que la generacin de nativos
digitales monopolice su tiempo libre adentro de casa y enchufado a un aparato. Este
comportamiento queda reflejado en un reciente estudio sobre los adolescentes y sus
hbitos culturales: la evidencia muestra all que ver televisin, escuchar msica y la
radio son las actividades que realizan ms asiduamente.
A esta altura, los resultados del relevamiento arecen en algunos casos obvios: el
99% de los adolescentes consultados mira tele ms de una vez por semana. Con esa
misma frecuencia de tiempo, el 96% escucha msica, el 80% est atento a la radio y el
55% se conecta a Internet. Dentro del ranking elaborado por la consultora TNS Gallup
para la Universidad de Palermo aparece como excepcin una actividad fuera del hogar

que incluso supera a Internet: hacer deportes, con el 64%. Aqu se incluyen los
deportes en la escuela, justifican los responsables del relevamiento.
Los datos quedan ms expuestos cuando se mira el fondo de esta tabla: ir al cine
(2%), al teatro y visitar exposiciones (1%), todas salidas por las que, generalmente, hay
que pagar, estn entre las menos frecuentadas. Tambin van a bailar menos de lo que
podra pensarse: de los adolescentes, slo uno de cada tres va al boliche semanalmente.
La cancha y el shopping tampoco son muy habituales.
Para Roxana Morduchowicz, directora del Programa Escuela y Medios, estos
ndices que, en principio haran pensar que la vida social y la relacin con el afuera est
en jaque, no hacen ms que demostrar la vida de hoy, pleno siglo XXI. Las pantallas
no anularon su vida social. En todo caso, generaron nuevas formas de sociabilidad
juvenil. La experta entiende que la prioridad para los jvenes sigue siendo relacionarse
con amigos, slo que ahora existen otros medios para hacerlo. Cuentan con ms
soportes para su vida social.
Cuando se les pregunta qu es un da divertido, responden salir con amigos. El
principal uso que hacen de la computadora es chatear. Y la funcin ms popular del
celular es mandar mensajes de texto a sus amigos, entiende. El estudio confirma lo que
dice Morduchowicz: el 68% de los consultados reconoce que chatear es lo que ms hace
en Internet.
Lo que hacen los jvenes no est ni bien ni mal, expresa las tendencias y
nuevas formas de integracin, sostiene desde Berln la sociloga argentina Liliana
Mayer, especialista en temas de juventud. No obstante, la experta opina: La
computadora, mediante su interactividad, permite la salida del mundo sin salir de casa.
En trminos del lazo social, lo fragmenta aun ms, ya que individualiza ms el ocio y
aumenta la reclusin en el tiempo libre. Pero en todo caso, es la sociedad la que no
motiva a los jvenes, que por eso no realizan actividades. Y no al revs.
(Disponible en: <http://www.clarin.com/diario/2010/03/15/um/m-02159751.htm>.
Acceso el 31/03/2010).
Questo 16
Basado en el texto, es correcto afirmar que los adolescentes argentinos
01) estn cada vez ms puertas adentro por falta de motivacin de la sociedad.
02) no se influencian por el desarrollo meditico.
04) siguen teniendo vida social, lo que pasa es que actualmente hay otras posibilidades
de hacerlo.
08) no van a museos porque sus padres no les llevan.
16) dicen no ir al cine pues les sale muy caro.
Questo 17
Sobre los resultados de la investigacin de que trata el texto, es correcto afirmar que
01) son resultados obvios.
02) los adolescentes prefieren chatear que salir con los amigos.
04) el cine, el teatro y las exposiciones son sitios menos frecuentados pues, a menudo,
hay que pagarlos.
08) el 2% de los consultados rechaza el cine.
16) un tercio de los adolescentes argentinos va a bailar semanalmente.
Questo 18

De acuerdo con el texto, es correcto afirmar que


01) el comportamiento de los adolescentes del siglo XXI se justifica simplemente
porque vivimos en la era de la pantalla.
02) entre la televisin y la Internet, la tele ocupa el primer lugar en los resultados del
relevamiento.
04) los deportes son la nica actividad que supera la Internet en la preferencia de los
jvenes.
08) la escuela es la responsable por la preferencia de los adolescentes por los deportes.
16) la prioridad de los adolescentes sigue siendo la comunicacin.
Questo 19
Respecto a los aspectos gramaticales y lxicos de la lengua espaola presentes en el
texto, es correcto afirmar que
01) en la frase La cancha y el shopping tampoco son muy habituales. (lneas 39-40),
es posible sustituir, indistintamente, tampoco por tambin.
02) para evitar la cacofona, la conjuncin y se ha sustituido por e en ... la msica
e Internet a las salidas. (lneas 1-2).
04) la palabra segn (lnea 2) es un caso de apcope.
08) el singular de estos ndices (lnea 42) es este ndice.
16) el verbo divertirse (ttulo) posee irregularidad -IE en presente de indicativo slo
en las tres personas del singular.
Questo 20
Segn el texto, hoy da los medios de comunicacin y los avances tecnolgicos
01) han provocado cambios en las costumbres de los adolescentes argentinos.
02) han hecho que chicos y chicas pasen un mal rato delante de la pantalla.
04) han estimulado mucho los usos del ordenador y del mvil.
08) han desarrollado el inters de chicos y chicas por ver la tele, escuchar msica y
navegar en Internet.
16) han mejorado la calidad de las escuelas.
Enem 2010 - Espanhol

1. De acordo com as informaes sobre aeroportos e estaes ferrovirias na Europa,


uma pessoa que more na Espanha e queira viajar para a Alemanha com o seu cachorro
deve:
a) consultar as autoridades para verificar a possibilidade de viagem;
b) ter um certificado especial tirado em outubro de 2004;
c) tirar o passaporte do animal e logo vacin-lo;

d) vacinar o animal contra todas as doenas;


e) vacinar o animal e depois solicitar o passaporte dele.

2. No ltimo pargrafo do fragmento sobre o bilinguismo no Paraguai, o autor afirma


que a lngua guarani, nas escolas, deve ser tratada como lngua de comunicao e no de
imposio. Qual dos argumentos abaixo foi usado pelo autor para defender essa ideia?
A. O guarani continua sendo usado pelos paraguaios, mesmo sem a escola e apesar dela.
B. O ensino mdio no Paraguai, sem o guarani, desmereceria todo o trabalho realizado e
as esperanas acumuladas.
C. A lngua guarani encontrou uma funcionalidade real que assegura sua reproduo e
continuidade, mas s isso no basta.
D. A introduo do guarani nas escolas potencializou a difuso da lngua, mas
necessrio que haja uma postura tica em seu ensino.
E. O bilingismo na maneira de ensinar o guarani tem causado estragos contra a lngua,
a cultura e a lealdade dos paraguaios ao guarani.
3. Em alguns pases bilnges, o uso de uma lngua pode se sobrepor outra, gerando
uma mobilizao social em prol da valorizao da menos proeminente. De acordo com
o texto, no caso do Paraguai, esse processo se deu pelo (a)
a) falta de continuidade de ensino do guarani nos programas escolares;

b) preconceito existente contra o guarani, principalmente nas escolas;


c) esperana acumulada na reforma educativa da educao mdia;
d) incluso e permanncia do ensino do guarani nas escolas;
e) continusmo do ensino do castelhano nos centros urbanos.

4. O texto jornalstico caracteriza-se basicamente por apresentar informaes dos mais


variados assuntos e seu ttulo antecipa o tema que ser tratado. Tomando como base o
fragmento, qual proposio identifica o tema central e poderia ser usada como ttulo?
a) Estilo de vida interfere no ganho de peso;
b) Estudo mostra expectativa de vida dos fumantes;
c) Pessoas que fumam podem se tornar anorxicas;
d) Fumantes engordam mais que no fumantes;
e) Tabagismo como fator de emagrecimento.

5. O texto publicitrio utiliza diversas estratgias para enfatizar as caractersticas do


produto que pretende vender. Assim, no texto, o uso de vrios termos de outras lnguas,
que no a espanhola, tem a inteno de:
a) atrair a ateno do pblico alvo dessa propaganda;
b) popularizar a prtica de exerccios esportivos;
c) agradar aos compradores ingleses desse tnis;
d) incentivar os espanhis a falarem outras lnguas;
e) enfatizar o conhecimento de mundo do autor do texto.

Eufemismo
Las plazas estn abarrotadas de hroes de bronce, literatos de mrmol y pensadores de
piedra, pero no hay ningn monumento al sagrado eufemismo que tantas y tantas

ventajas nos procura. Aprendimos a no decir la verdad descarnada. Aprendimos a tener


una doble cara, una doble vida, una moral y otra inmoral, una cvica y otra rebelde, dos
personalidades a menudo irreconciliables.
Tenemos la creencia de que sin una pizca de hipocresa y una disposicin favorable
hacia el otro, el mundo sera invivible, una selva terrorfica de dardos veraces en busca
de la primera vctima inocente.
De entrada parece que el lenguaje tenga culpa de todo, pues decimos lo que decimos en
parte porque lo hemos heredado con una carga patriarcal, sexista, racista, supersticiosa,
legitimadora del poder entre otras cosas. As que cuando nombramos algo no slo lo
sealamos, lo evocamos y lo definimos (que en eso radica la funcin de la lengua), sino
que tambin lo discriminamos o lo estigmatizamos a gusto de nuestra ideologa.
Las minoras arremeten contra los abusos del lenguaje y el movimiento "polticamente
correcto" lucha para corregir las discriminaciones que mantiene nuestra cultura
dominante. Sin embargo, no por decir persona madura en vez de viejo cambia con ella
la realidad o la discriminacin que sufren estas personas en una sociedad profundamente
desigual. Porque antes que las palabras est el pensamiento del individuo del cual
aquellas brotan, pero este pensamiento no es nada sin la mentalidad colectiva donde se
sostiene. Por eso, fijmonos ms en la carga que pone el individuo y los grupos en las
palabras y no en las palabras mismas que en ltima instancia son neutras.
Es cierto que cambiando una palabra por otra cambiamos el acento desvalorizador que
aquellas tenan, pero tambin hemos de tener en cuenta que aadimos a las nuevas los
acentos y los intereses del grupo, minoritario o mayoritario, que reclama la correccin.
El poder suele tener un punto ciego, una voluntad de dominacin aunque se rodee de
mensajes populares y humanistas, y una de sus mejores armas es el discurso que parece
decir algo pero no dice nada. Discurso que confunde porque da la impresin de querer
agradar a todos pero, a decir del ojo atento, lo que quiere es atontar para desviar la
mirada de lo verdaderamente importante, aquello que evidentemente no se puede
destapar.
Nuestros odos estn acostumbrados a esa capa de irrealidad al que nos tiene
acostumbrados el mensaje poltico, militar y econmico, de tal manera que cuando un
pas hegemnico invade otro pas es por el nuevo orden mundial y a favor de la
democracia. Si la empresa te despide, debemos decir reajuste de recursos humanos.
Cuando leo el peridico o veo la televisin intento leer entre lneas y estar atento para
distinguir lo que se dice de lo que realmente se quiere decir y poder adivinar lo que no
se dice pero que es lo verdaderamente importante. Por otro lado, cuando hablo o escribo
utilizo las mismas palabras que todos utilizamos (de alguna manera nos hemos de
entender), pero procuro ver la intencin que las mismas palabras embeben o el corazn
que late entre ellas.
El tesoro del lenguaje es que nos hace vivir mundos inimaginables y adems nos
permite comunicarlos, pero el peligro de ste es que nos eleva por encima de la realidad
dejndonos ante el abismo que nos separa de ella. El silencio es su terapia y por eso, a

veces, recuerdo un dicho sabio de los indios norteamericanos que dice: escucha o tu
lengua te volver loco!
JULIN PERAGN
www.concienciasinfronteras.com
1. El eufemismo consiste en la sustitucin de una palabra o frase por otra, para
disimular la vulgaridad o gravedad de la original.
Para Julin Peragn, la construccin y mantenimiento del eufemismo se debe
principalmente
(A) herencia de las minoras
(B) ideologa del individuo
(C) lucha de los polticos
(D) apoyo de la sociedad
2. Segn el texto, sera justo erigir un monumento al eufemismo ya que su uso se
impone como modo de vivir en sociedad.
Ese uso del eufemismo hace que las relaciones entre las personas sean consideradas
como:
(A) confiables
(B) dinmicas
(C) viables
(D) slidas
3. En el penltimo pargrafo, el autor utiliza la primera persona de singular en lugar de
la forma nosotros.
Se puede comprender que ese cambio tiene como objetivo principal:
(A) sugerir un consejo
(B) explicar un concepto
(C) constatar un equvoco
(D) contradecir un punto de vista
4. Nuestros odos estn acostumbrados a esa capa de irrealidad
Esa idea de accin habitual presente en la locucin verbal subrayada tambin se la
encuentra en el siguiente fragmento:
(A) El poder suele tener un punto ciego,
(B) debemos decir reajuste de recursos humanos.
(C) y poder adivinar lo que no se dice
(D) y adems nos permite comunicarlos,
5. Peragn nos trae una reflexin respecto al contenido transmitido por los medios de
comunicacin.

Para el autor, ante el lenguaje de los medios de comunicacin, la gente debe tener la
actitud de:
(A) apata
(B) perplejidad
(C) desconfianza
(D) aburrimiento
6. escucha o tu lengua te volver loco!
En el dicho de los indios, el conectivo subrayado establece una relacin de sentido que
se
puede comprender como:
(A) duda
(B) exclusin
(C) alternancia
(D) explicacin
Texto
Trabajadoras o siervas?
Las empleadas de hogar se rigen por regulaciones desfasadas que las hacen vulnerables
a la explotacin - De 700.000 hogares con servicio, menos de 300.000 cotizan - El
Gobierno negocia frmulas para incluirlas en el rgimen general
Maria R. Sahuquillo
Rosa [...] es empleada domstica interna y cobra menos de 600 euros al mes. No llega al
salario mnimo interprofesional. Esta paraguaya de 45 aos cotiza a la seguridad social a
travs del rgimen especial de empleados del hogar. Segn esta ley, no tiene prestacin
por desempleo. Y, si se pone enferma, no disfruta de una baja pagada hasta que lleve 29
das en cama; cuando las bajas retribuidas de los trabajadores del rgimen general son a
partir del tercero, y en los autnomos, del cuarto (aunque con el 60% del salario).
Un rgimen anacrnico que les impide cotizar si trabajan menos de 72 horas al mes y
que ni siquiera les garantiza un contrato de trabajo escrito que puedan blandir para hacer
valer sus derechos en caso de conflicto con los empleadores. Es una regulacin de 1985
que sita a las miles de empleadas domsticas que hay en Espaa. Ahora negocian con
el Gobierno la frmula de inclusin en el rgimen general de trabajadores.
"Las empleadas de hogar se enfrentan a una discriminacin clara", critica Begoa San
Jos, del Frum de Poltica Feminista. "Son una fuerza vital en la sociedad. Mueven un
engranaje de cuidados sin el que las cosas no funcionaran. A pesar de eso, estn en una
situacin de enorme desventaja. Mucho tiene que ver con esa discriminacin que la casi
totalidad sean mujeres. Si los chferes domsticos estn en el rgimen general de
trabajadores, por qu las empleadas domsticas no? Eso debe cambiar", exige. La

reforma no ser pequea. Ms de 700.000 hogares espaoles emplean a una trabajadora


domstica, segn la Encuesta de Poblacin Activa del INE. Sin embargo, solo 296.067
personas estn afiliadas al rgimen especial de empleo de hogar de la Seguridad Social.
Un dato, en comparacin, muy bajo incluso a pesar de que hay empleadas que
trabajan en varias casas , que revela que muchas de estas personas ejercen al margen
de legalidad. Y que, por tanto, carecen de derechos.
"No es solo que la legislacin sea mala, sino que adems se incumple
sistemticamente", apunta la economista Amaia Prez Orozco, investigadora en temas
de gnero. El rgimen de empleo de hogar fija un tiempo de descanso de 36 horas a la
semana, aunque no obligatoriamente continuadas, para estas empleadas. Una clusula
que no siempre se practica. Muchas internas, como Rosa, solo tienen una tarde libre
cada siete das. La ley fija que no pueden cobrar menos del salario mnimo
interprofesional (641 euros al mes). Pero si esa cantidad se calcula anualmente, cobran
menos, ya que tienen dos pagas extras anuales de 15 das, frente a las dos pagas de mes
completo del resto de trabajadores.
Las cosas, aunque poco a poco, comienzan a cambiar. Hace unos meses el Ministerio de
Trabajo elimin uno de los puntos ms escandalosos del rgimen. La opcin de que se
les descontase el 45% del salario en concepto de manutencin y alojamiento (si era el
caso). Una frmula que, segn Octavio Granado, secretario de Estado de Seguridad
Social, provocaba situaciones "verdaderamente penosas". No obstante,
muchas empleadas la siguen sufriendo.
(Disponible en <http://www.elpais.com/articulo/sociedad/Trabajadoras/si
ervas/elpepusoc/20110601elpepisoc_1/Tes>. Acceso en 01/06/11 a las 15h10min)
Questo 36
Seale lo que es correcto, segn el texto.
01) El texto evidencia que en Espna no llega a un 50% el nmero de trabajadoras del
hogar que tienen asegurados sus derechos laborales.
02) Al ponerse enferma, una empleada del hogar en Espaa recibir una paga ms baja
que la corriente.
04) Las empleadas del hogar prefieren recibir menos que el sueldo mnimo
interprofesional porque compensan con las dos pagas extras de 15 das.
08) El texto pone en evidencia que hay una propuesta para el gobierno intentar
equiparar los derechos de las empleadas con los de los dems trabajadores.
16) Uno de los indicios que demuestra que la ley en Espaa adolece de ajustes es el
hecho de la paraguaya pagar las cuotas de seguridad social.

Questo 37
Assinale a(s) alternativa(s) correta(s), segundo o texto.
01) A escrita correta em espanhol do numeral 641 (linha 60) seiscientos cuarenta y
uno.
02) Ao escrever o numeral 29 (linha 9) em espanhol, devem-se utilizar duas palavras:
veinte nueve.
04) Se uma empregada tiver um desconto no salrio, devido ao fato de morar no
trabalho, poder ganhar menos de 60% do salrio mnimo interprofissional dos
trabalhadores inscritos no regime geral espanhol.
08) Se fosse utilizada a expresso tercer da no lugar de tercero (linha 12), no
haveria alterao no significado da orao.
16) Os trabalhadores que fazem parte do regime geral espanhol no podem receber mais
de 350 euros, caso fiquem doentes.
Questo 38
Con relacin al vocabulario, seale lo que es correcto.
01) Por el trmino prestacin (lnea 7), se entiende que no hay beneficio social para
las empleadas del hogar.
02) El vocablo siguen (lnea 76) indica que hay una vulneracin en los derechos de
las internas.
04) Al referirse al trmino servicio, el subttulo del texto alude a la accin que
mujeres y hombres realizan en los trabajos hogareos.
08) En la lnea 7, hay un verbo tiene, cuyo sujeto se presenta en la oracin anterior.
16) En el subttulo, el vocablo vulnerables (subttulo) es un adjetivo que tiene la
funcin de caracterizar las internas.
Questo 39
Assinale a(s) alternativa(s) correta(s), de acordo com o texto.
01) O vocbulo Mucho (linha 31) refere-se descrio de que as empregadas
domsticas so uma fora vital na sociedade.
02) A empregada domstica deve descansar ao menos 36 horas por semana, devendo ser
tais horas de modo ininterrupto. Esta legislao fixada pelo governo espanhol.
04) O adjetivo penosas (linha 75) pode ser substitudo pelo adjetivo espinosas, sem

alterar o sentido da frase.


08) A expresso Seguridad Social (linhas 42-43) o mesmo que segurana social em
portugus e refere-se ao auxlio dado pelo governo aos trabalhadores.
16) O pronome possessivo sus (linha 18) poder ser substitudo por suyos, quando
o substantivo derechos (linha 18) for citado anteriormente ao possessivo.
Questo 40
Com referncia aos elementos lingusticos, correto dizer que
01) blandir (linha 17) um verbo da 2. conjugao, que pode ser substitudo por
escribir sem mudar o sentido da orao.
02) os verbos impide (linha 14) e garantiza (linha 16) pertencem 2. e 1.
conjugao, respectivamente.
04) a expresso al margen (linhas 46-47) um exemplo de contrao, pois margen
um substantivo masculino.
08) na linha 30 o termo eso um demonstrativo masculino.
16) as palavras rgimen (linha 5), comparacin (linha 43) e sistemticamente
(linha 50) so exemplos de palavras esdrujla, aguda e sobresdrjula.
ESPANHOL
Texto
La vida moderna es un txico
Emilio de Benito
Los afectados por sensibilidad qumica mltiple luchan para que se reconozca su
dolencia - Productos diarios como la colonia son una agresin para ellos Espacio libre
de aromas. Apaga tu mvil. No lleves colonia, desodorante y, si es posible, lava la
ropa con bicarbonato. Las instrucciones para visitar la Fundacin Alborada en Brunete
(a unos 30 kilmetros de Madrid) hacen temer que se trate de un espacio inhspito. Una
burbuja, que es la imagen a la que se asocia a las personas con sensibilidad qumica
mltiple, una enfermedad que les hace reaccionar de manera exagerada y
desagradable ante las sustancias generadas por la vida moderna. Nada ms lejos de la
realidad. La finca El Olivar est aislada, s, pero, presidida por una encina milenaria,
parece ms bien una residencia de verano que un centro sanitario.
Era una residencia para tratar adicciones, pero hace dos aos, cuando volv de Dallas,
le cambiamos la utilidad, explica Pilar Muoz-Cavero (Madrid, 1955), mdica y
afectada. En el ao y medio que lleva en funcionamiento, han recibido a unas 200

personas. Algunas vienen solo por informacin, otras pasan aqu el da, de 9.00 a
17.00, desintoxicndose, cuenta.
Desintoxicarse es la palabra que Muoz- Cavero ms repite. En las ltimas dos
dcadas se han introducido ms de 100.000 sustancias. Pesticidas, solventes orgnicos,
metales pesados, muchos productos de higiene son objetivamente dainos, pero adems
hay personas a las que les afectan ms. Por algn motivo, no las eliminan, sino que las
acumulan. El cuerpo se sobrecarga. Es la vida moderna la que resulta txica. Los
avances estn bien, pero para algunos son demasiado, explica. El resultado, tal y como
lo vivi ella, es malestar, prdida de memoria, dolor de huesos, de cabeza, muscular,
cansancio, irritabilidad.
El sndrome es diverso. Unas veces aparece poco a poco, como una carga que vas
empezando a notar, hasta que una exposicin a un txico lo desencadena, cuenta
Muoz-Cavero. Otras veces surge tras una exposicin aguda a una sustancia, como
unas limpiadoras que entraron en un sitio donde acababan de fumigar. Como el cuerpo
va acumulando los txicos, nunca se sabe cul va a ser la sustancia que sea la gota que
colma el vaso, describe.
Pero la buena noticia para los afectados que se calcula que en mayor o menor grado lo
son un 15% de la poblacin es que hay tratamiento. Nos viene gente de todas las
edades. Hemos tenido hasta nios de ocho o nueve aos. Como muchos de los txicos
son liposolubles, pasan al feto a travs de la placenta, as que cuando nacen ya vienen
con cierta carga, afirma. Tambin aumentan los hombres afectados.
El tratamiento no es fcil. En su caso, pas un ao encerrada entre cuatro paredes.
No poda ni beber agua, porque la rechazaba. Estaba ms muerta que viva. Aislarse
es, por eso, el primer paso. Evitar la exposicin.
Esta es la fase ms complicada. Hay que cambiar los hbitos, aprender a consumir,
dice. Los alimentos, mejor si son orgnicos; la ropa no puede ser acrlica, mejor seda o
algodn. Lana tambin, aunque hay personas que no la aguantan. Todo se ve afectado:
la casa (evitar estar en el entorno de fbricas, de sitios muy contaminados, de emisores
de radiacin). Por eso nos llaman los nmadas, o, tambin, los centinelas de la vida,
porque somos como los canarios en las minas de carbn, que eran los primeros en
detectar el gris, relata.
(Disponible en http://www.elpais.com/articulo/salud/vida/moderna/toxico/
elpepusal/20110531elpepisal_1/Tes acceso el 01/6/2011 a las 15h20min)
Questo 36
Assinale a(s) alternativa(s) correta(s).

01) O artigo el que aparece em el da (linha 25) e El cuerpo (linha 35) seguem a
mesma regra quando usados em el cansacio e El irritabilidad.
02) De acordo com o texto, a primeira atitude que se deve ter para diminuir os riscos de
alergias no se expor a possveis substncias txicas.
04) O texto demonstra que ainda no se reconhece a alergia e suas consequentes
infeces com o status de real doena.
08) Os sintomas da sensibilidade qumica nem sempre so visveis, como o caso de
dor de cabea ou no corpo. Alm disso, podem aparecer de forma diferenciada, sendo
mais ou menos intensa.
16) O termo adicciones (linha 19) pode ser substitudo por enfermedades sem
mudana de sentido.
Questo 37
Seale la(s) alternativa(s) correcta(s).
01) La oracin Por algn motivo, no la eliminan, sino que las acumulan (lneas 34-35)
puede ser sustituida por: por alguna razn no la eliminan, por el contrario, las
acumulan.
02) El pronombre le (lnea 20) est relacionado con una informacin presentada
anteriormente.
04) La oracin La finca El Olivar est aislada, s, pero, presidida por una encina
milenaria ... (lneas 13-15) puede ser sustituida por La hacienda est alejada, s, mas,
presidida por una cultura milenaria ....
08) El vocablo sitio (lnea 49) se refiere a un lugar que los campesinos haban
desinfectado.
16) Las oracines Aislarse es, por eso, el primer paso y Evitar la exposicin (lneas
67-68) son ejemplos de prescripciones.
Questo 38
Com referncia aos elementos lingusticos, correto afirmar que
01) o verbo Apaga (linha 1) est em seu modo imperativo afirmativo e, em seu modo
imperativo negativo, seria no apagues.
02) o numeral dos (linha 19) sofre flexo de gnero. o mesmo caso do numeral
un, o qual com a flexo de gnero ficar una como em una residencia... (linha
16).

04) o artigo los, como em Los avances (linhas 36-37), ser usado tambm para o
substantivo informacin (linha 25), quando estiver no plural.
08) o plural de imagen (linha 8), realidad (linha 13) e exposicin (linha 45) ,
respectivamente, imgenes, realidades e exposiciones.
16) o pronome su que aparece em En su caso (linha 63) pode ser substitudo pelo
pronome tu, sem alterar o sentido da frase.
Questo 39
Seale la(s) alternativa(s) correcta(s).
01) Tanto los centinelas como los canarios tienen la funcin de detectar el gris.
02) Algunas (lnea 24) se refiere a personas (lnea 24). Si el vocablo personas
fuera sustituido por enfermos, el indefinido que le acompaara sera algns.
04) El vocablo burbuja (lnea 7) puede ser usado en la expresin burbuja
tecnolgica con el mismo sentido del texto.
08) El trmino tras (lnea 47) quiere decir atravs en portugus.
16) El trmino encerrada (lnea 64) quiere decir trancada en portugus.
Questo 40
Seale lo que es correcto.
01) El texto demuestra que hay que repensar el empleo de algunos productos comunes
de uso dirio porque pueden daar a la salud de nios y de diferentes hombres.
02) El texto trata del problema de la imagen esteriotipada que las personas suelen tener
de los afectados por sensibilidad qumica mltiple.
04) El trmino burbuja (lnea 7) est empleado en el texto en el sentido figurado.
08) Las instrucciones para visitar la Fundacin Alvorada hace uno pensar que es un
lugar inhabitable.
16) El centro de desintoxicacin creado por la mdica afectada, aunque haga a la gente
temer por las instrucciones que se presentan, tiene un aspecto de residencia veraniega.
Un cero en oratoria
Los adolescentes carecen de habilidad para expresarse en pblico La receta de los
expertos pasa por potenciar los debates en la escuela y la Universidad
Elisa Sili

Con independencia de saberse la leccin o no, cuando a un adolescente le llaman a la


pizarra le embarga el pavor escnico. Es ms que habitual que el reo esquive entonces la
mirada, juegue con el boli, cruce los brazos, se atuse el pelo nerviosamente y no
encuentre las palabras adecuadas para hacerse entender [...]. Veredicto: un cero en
oratoria. La mayora de los espaoles de la quema no se salvan tampoco los adultos
no controla ninguno de los tres tipos de lenguaje: verbal (palabra), gestual o paraverbal
(voz). Cuesta hasta levantar la mano para preguntar en clase. "Nos puede el sentido del
ridculo y no hay que tenerlo.
Hablar es algo natural y hacerlo delante de mucha gente no cambia las cosas", asegura
la periodista Mnica Prez de las Heras, que imparte cursos a profesionales. "No
gestionamos bien el lenguaje, delegamos en el subconsciente. Decimos una cosa de
palabra pero, a lo mejor, lo contrario con los gestos", prosigue.
"Hay que aprender a controlar la inteligencia emocional desde pequeos. Por ejemplo,
en la guardera, trabajando la empata. En Estados Unidos, a los 11 aos los nios
exponen en clase, debaten y cuando llegan a la Universidad ya estn preparados",
sostiene Prez de las Heras, autora de Ests comunicando? y El secreto de Obama.
"Los nios ven como un castigo salir a la pizarra y, por eso, para habituarles, deberan
de explicarse delante de ella desde los tres aos", aade Miguel Gmez- Aleixandre,
profesor del club de debate del CEU-San Pablo. Manuel Snchez-Puelles, ganador de
varios concursos hablados, tambin apuesta por la edad temprana: "Los nios no tienen
vergenza y hay que aprovechar ese momento para que expresarse en pblico les
parezca una cosa natural".
En Italia y Francia abundan los exmenes orales en secundaria y pasan una prueba en
Selectividad. En Espaa, a partir del prximo ao, existir un examen oral en lengua
extranjera para acceder a la Universidad, mientras los Gobiernos autnomos planean
organizar campeonatos de debate. Tmidamente empiezan a celebrarse tambin en
colegios, como el Balder (Las Rozas, Madrid) cuyos alumnos de 5 y 6 de primaria
compitieron en su octava edicin con tres escuelas. El tema a discutir: reciclamos todo
lo que necesita nuestro medioambiente? Cada equipo lo formaba un capitn, un
documentalista y tres oradores. "Una cosa es repetir como un papagayo lo que has
aprendido en un examen y otra convencer al pblico construyendo un discurso que se va
cambiando si se necesita", diferencia Adolfo Lucas, coordinador de las mesas de debate
en la Universidad Abat Oliba CEU y autor de El poder de la palabra. "Desde el
momento en el que el nio tiene una base de lectura y escritura, cuando uno es capaz de
expresarse, debe aprender persuasin".
Oratoria no es una asignatura obligatoria en las universidades espaolas, "pero lo ser a
medio plazo porque Bolonia impulsa estas habilidades y capacidades", asegura GmezAleixandre, coordinador del campeonato Inter CEU. Cada vez ms universidades
ofrecen clases complementarias. Entre esa red tambin se organizan torneos en los que
se enfrentan conjuntos de dos a seis personas. "Te proponen dos temas y tienes 20 das

para preparar la postura a favor y en contra. Un minuto antes, te enteras de qu vas a


debatir y en qu lado ests", cuenta Snchez-Puelles.
(Disponible en <http://www.elpais.com/articulo/educacion/cero/oratoria/e
lpepuedu/20110516elpepiedu_1/Tes>. Acceso el 01/6/11 a las 15h35min)
Questo 27
En lo que se refiere al sentido de los vocablos presentes en el texto, es correcto decir
que
01) la palabra nios (lnea 32) es utilizada para referirse a los espaoles en general.
02) en el fragmento Oratoria no es una asignatura obligatoria en las universidades
espaolas, pero lo ser a medio plazo... (lneas 70-72), se puede entender que los
interesados deben apuntar su nombre desde ya en la lista de los aspirantes al estudio.
04) en la expresin clases complementarias (lnea 76), el adjetivo complementarias
demuestra que las universidades empiezan a ofrecer ms salones de aula.
08) el vocablo tampoco (lnea 10) es un adverbio que tiene un sentido negativo.
16) el trmino cuyos (lnea 53) es un mecanismo lingustico que demuestra que los
alumnos estudian en el colegio Balder.
Questo 28
En relacin con la lectura del texto, es correcto afirmar que
01) un ejemplo apuntado sobre cmo controlar la inteligencia emocional es el trabajo
que hacen con los guardias.
02) el texto es relevante por presentar instrucciones para que uno aprenda a hablar bien
en pblico.
04) el objetivo del texto es la de reforzar la necesidad que tienen las escuelas y
universidades espaolas de preparar a sus alumnos para expresarse en pblico.
08) Snches-Puelles tambin defiende la necesidad de comenzar a preparar a los
alumnos para hablar en pblico desde la niez.
16) Mnica Peres afirma que hablar delante del pblico ayuda a gestionar mejor el
lenguaje.
Questo 29
A partir del texto, es correcto afirmar que
01) debido al carcter reservado del espaol tanto adultos como adolescentes tienen
dificultad en oratoria.

02) est previsto el desarrollo de la capacidad de oratoria de los alumnos de las


Universidades Espaolas.
04) los adolescentes espaoles se sienten absorbidos por el terror si les invitan a hacer
un teatro.
08) arreglarse el pelo es una reaccin usual del estudiante que se pone nervioso y no
consigue hablar con elocuencia.
16) una de las razones por las cuales al espaol no le gusta exponerse en clase es el
sentido de ridculo que le domina.
Questo 30
En cuanto a los mecanismos textuales extrados del texto, es correcto afirmar que
01) la locucin a lo mejor (lnea 22) podra ser sustituida por tal vez sin cambiar el
sentido de la oracin.
02) el vocablo hasta (lnea 13) puede ser traducido al portugus por ainda.
04) la palabra Veredicto (lnea 8) se refiere a la mala relacin que tienen los espaoles
con el lenguaje.
08) en la lnea 4, el trmino reo es utilizado para demostrar que los alumnos espaoles
son antisociales.
16) la preposicin desde (lnea 25) est relacionada con el tiempo en que ha de
empezar a preparar a los alumnos para hacer el buen uso del lenguaje.

You might also like